Med-Surg Chapter 28

Pataasin ang iyong marka sa homework at exams ngayon gamit ang Quizwiz!

The nurse provides teaching to a patient with asthma who has been advised to use nebulization. What should the nurse include in the instructions about nebulization?

Sit in an upright position during the treatment

The nurse is teaching a patient how to use a hand-held nebulizer. Which guideline is correct?

Sit in an upright position during the treatment.

The nurse determines that a 61-year-old patient with chronic bronchitis has a nursing diagnosis of "impaired gas exchange" after noting an oxygen saturation of 88%. What is an appropriate intervention to add to the care plan?

Sit the patient upright in a chair leaning slightly forward

The nurse determines that a 61-year-old patient with chronic bronchitis has a nursing diagnosis of "impaired gas exchange," after noting an oxygen saturation of 88%. What is an appropriate intervention to add to the care plan?

Sit the patient upright in a chair leaning slightly forward

A patient has had chronic obstructive pulmonary disease (COPD) for three months. Further assessment reveals that forced expiratory volume in one second (FEV1) is 65 percent. Which treatment option would be most appropriate for this patient?

A patient who has had chronic obstructive pulmonary disease for three months and who has an FEV1 of 65 percent would require treatment with long-acting bronchodilators such as albuterol or ipratropium. These drugs can be used as monotherapy, but combining them improves the effect and decreases the risk of side effects. Salmeterol monotherapy is less effective than combination therapy of albuterol and ipratropium because of its lower efficacy and higher risk for side effects. Budesonide is an inhaled corticosteroid, which should not be used as monotherapy in patients with COPD because of its side effects. Budesonide and formoterol combination is used for COPD in patients with FEV1 of less than 60 percent.

A child with a nonproductive cough states, "I am having trouble breathing." What action should the nurse take to reduce the severity of breathlessness in the child? 1 Assist the child to lie in supine position 2 Instruct the child to bend forward slightly 3 Suggest that the child walk for 30 minutes 4 Instruct the child to take short, quick breaths

Answer:2

Which nursing intervention will the nurse include in the teaching plan for a patient with difficulty in breathing due to low oxygen level in the blood? 1 Suggest that the patient breathe slowly with an open mouth. 2 Advise the patient to refrain from sexual activity after eating. 3 Ask the patient to plan sexual activity during the late afternoon. 4 Suggest that the patient choose the missionary position during intercourse

Answer:2

The nurse is educating the patient on oxygen use for the home. The patient asks the nurse "Does this mean that I will not be able to go anywhere?" The most appropriate response by the nurse is:

Encourage the patient to continue normal activity and travel plans

The nurse is caring for a patient with an oxygen saturation of 45 percent of personal best who stops breathing while sleeping and has a tendency to sleep during the day. Which intervention by the nurse will help the patient most while recovering?

Encourage the patient to use typed messages to communicate.

What is the best method to prevent oral infection while the patient is taking fluticasone?

Rinse the mouth with water after the second puff of medication

The nurse is assisting a patient to learn self-administration of beclomethasone, two puffs inhaled every six hours. What should the nurse explain as the best way to prevent oral infection while taking this medication?

Rinse the mouth with water following the second puff of medication

A four-year-old child with cystic fibrosis (CF) experiences a cough associated with viscous, purulent, and greenish sputum. The nurse expects that which organism will be found in the patient's sputum?

Staphylococcus aureus

A patient receives a prescription for omalizumab. What medication route of administration and schedule will the nurse include in the information provided to the patient?

Subcutaneously, every two to four weeks

When caring for a patient with chronic obstructive pulmonary disease (COPD), the nurse identifies a nursing diagnosis of imbalanced nutrition: less than body requirements after noting a weight loss of 30 lb. Which intervention should the nurse add to the plan of care for this patient?

Because the patient with COPD needs to use greater energy to breathe, there often is decreased oral intake because of dyspnea. A full stomach also impairs the ability of the diaphragm to descend during inspiration, thus interfering with the work of breathing. For these reasons, the patient with COPD should eat six small meals per day, taking in a high-calorie, high-protein diet, with non-protein calories divided evenly between fat and carbohydrate. Prescribing fruits and fruit juices, teaching the patient to use frozen meals at home, and providing a high-calorie, high-carbohydrate, nonirritating, frequent feeding diet will not increase the patient's caloric intake.

Which cells cause inflammation in a patient with an inflamed airway and productive cough who has Pseudomonas aeruginosa infection?

Neutrophils

Which parameter should the nurse consider while assessing a patient with cystic fibrosis (CF)?

Abundant bronchial mucus

The nurse recalls that which type of asthma involves an acute asthma attack triggered by an upper respiratory infection? 1 Intrinsic 2 Allergic 3 Extrinsic 4 Emotional

Answer:1

A patient requires oxygen administration in low concentrations of 24% at 1 L/min for a long duration. Which device is the most appropriate for this patient?

Nasal cannula

A patient needs to receive oxygen at a 28% concentration. The nurse will set up which oxygen delivery device?

Venturi mask

The nurse recalls that the steps that lead to airway remodeling occur in what order?

-Inhalation of animal dander -Degranulation of mast cells -Release of inflammatory mediators -Cellular infiltration -Neuropeptides release

The patient with asthma asks the nurse, "How will I know when my inhaler is empty?" What is the best response by the nurse?

"You need to keep track of how many puffs you have used and how many puffs are available."

The patient has a prescription for albuterol 3 mg by nebulizer. Available is a solution containing 24mg/mL. How many mL should the nurse administer?

0.125 mL

The nurse is assessing a patient who is having an acute asthma attack. Which nursing intervention is the priority for this patient?

Administering an albuterol bronchodilator

A patient develops vascular congestion, edema, and breathlessness within 30 minutes of the consumption of food containing tartrazine. The nurse identifies that which treatment may exacerbate the patient's condition?

Administration of intravenous corticosteroid

A patient with asthma has a body temperature of 102° F and produces purulent sputum. The nurse anticipates that which drug will be prescribed?

An antibiotic

A patient presents to the emergency department with sudden-onset wheezing and coughing with progressive respiratory distress. What condition or diagnosis does the nurse recognize?

An asthma attack

A 45-year-old man with asthma is brought to the emergency department by automobile. He is short of breath and appears frightened. During the initial nursing assessment, which clinical manifestation might be present as an early manifestation during an exacerbation of asthma?

Anxiety

A patient with asthma is admitted in the emergency department with severe dyspnea and is unable to speak. The nurse finds that the patient looks drowsy and confused. Which other finding does the nurse expect?

Arterial blood gas deterioration

What is the most common sign during an initial assessment that alerts the nurse that the patient has chronic obstructive pulmonary disease?

Barrel chest

What is the most appropriate time to obtain peak flow readings with the best peak flow number?

Between noon and 2 PM

The nurse caring for a patient with diabetes mellitus and chronic obstructive pulmonary disease (COPD) on oral prednisone will monitor which parameter regularly?

Blood sugar

The nurse anticipates that which treatment therapy will be prescribed for a patient with massive hemoptysis?

Bronchial artery embolization

The nurse identifies that which treatment strategy will be beneficial for a patient that has a hemoglobin level of 7.8 mg/dL, inflammation, hemoptysis, and elastic and muscular structural changes to the bronchial wall?

Bronchial artery embolization

The nurse anticipates that which treatment therapy will be prescribed for a patient with massive hemoptysis?

Bronchial artery embolization helps treat the patient with massive hemoptysis. Chest tube drainage helps the patient with pneumothorax. The patient with cor pulmonale or hypoxemia may require home oxygen therapy. Sclerosing of the pleural space benefits the patient with pneumothorax.

Which corticosteroid can be administered directly through inhalation?

Ciclesonide

What is an autosomal recessive, multisystem disease characterized by altered function of the exocrine glands?

Cystic fibrosis (CF)

The nurse is caring for a patient with asthma. The patient's forced expiratory volume in one second is 70 percent. What does the nurse infer regarding how often the patient's symptoms occur?

Daily, particularly in the morning

A patient with asthma is prescribed ipratropium bromide. The nurse recognizes that the patient may develop what side effect of the medication?

Dry mouth

patient with asthma is prescribed ipratropium bromide. The nurse recognizes that the patient may develop what side effect of the medication?

Dry mouth

The nurse determines that a patient is not experiencing the beneficial effects of ipratropium after noting which finding in the patient?

Expiratory wheezing

A patient with allergic asthma has been prescribed omalizumab. The patient requests that the medication be administered at home for convenience. Which nursing action is appropriate in this case?

Explain to the patient that the medication should be administered strictly at the clinic

Before administering ipratropium bromide, the nurse must confirm that the patient does not have which comorbidity?

Glaucoma

What are the structural changes that occur during chronic inflammation? Select all that apply

Hypersecretion of mucus Progressive loss of lung function Proliferation of new blood vessels

A patient who is a chronic smoker is diagnosed with chronic obstructive pulmonary disease (COPD). What causes the loss of elastic recoil in the patient's lungs and the destruction of alveoli?

Imbalance of protease and antiprotease ratio

The nurse is assessing a patient with asthma who has been diagnosed with a severe and life-threatening exacerbation. What findings would the nurse find?

Increased CO2 level

Which change occurs during cystic fibrosis?

Increased inflation of lungs

When teaching the patient with bronchiectasis about manifestations to report to the health care provider, which manifestation should be included?

Increasing dyspnea

The nurse is caring for the patient with chronic obstructive pulmonary disease (COPD). The patient complains of difficult breathing. What action by the nurse is most appropriate?

Initiate oxygen administration at low flow rates

What is a priority nursing assessment for a 38-year-old patient experiencing an acute asthma exacerbation?

Inspection of the chest wall

Which intervention is beneficial to a patient with chronic obstructive pulmonary disease (COPD)?

Limiting fluids during mealtimes

What complication does the nurse expect in a child with chronic pulmonary disease who is diagnosed with α1-antitrypsin (AAT) deficiency?

Liver disease

A patient with dyspnea and hypoxemia has received an initial nebulized short-acting β2-adrenergic agonist (SABA) with ipratropium treatment. The patient's forced expiratory volume in one second is 60 percent, and the peak flow is less than 25 percent of personal best. The nurse anticipates that which medication will be administered?

Magnesium sulfate

A patient who has been taking low-dose inhaled corticosteroids along with long-acting ß2-agonists for three months is assessed for control of asthma. The patient's laboratory reports reveal inadequate control of asthma. Which treatment does the nurse expect to be prescribed?

Medium dose inhaled corticosteroids and long-acting ß2-agonist

When should a nurse schedule postural drainage for a patient who has chronic obstructive pulmonary disease (COPD)?

One hour before a meal

Prolonged exposure to a high level of oxygen leads to pulmonary damage caused by what?

Oxygen toxicity

Which assessment finding does the nurse expect when caring for a patient with asthma?

PaCO2 of 30 mm Hg

When teaching the patient with cystic fibrosis about the diet and medications, what is the priority information to be included in the discussion?

Pancreatic enzymes and adequate fat, calories, protein, and vitamins are needed

A patient with cystic fibrosis (CF) experiences oily, irregular bowel movements and has a fat-soluble vitamin deficiency. The nurse anticipates what prescription?

Pancrelipase before each meal

The nurse is caring for the patient with cystic fibrosis (CF). Which is the most important factor for the nurse to consider?

Patients with CF often survive into their 40s.

When teaching a patient with chronic obstructive pulmonary disease (COPD) about using pursed-lip breathing (PLB), what is the most important instruction?

Perform PLB before, during, and after activities that cause shortness of breath

A nurse is caring for a patient who is having an acute asthma attack. Which interventions should the nurse question? Select all that apply.

Prednisone 10 mg, PO X1 stat Administration of blood products to increase oxygen saturation

When caring for a patient with chronic obstructive pulmonary disease (COPD), the nurse identifies a nursing diagnosis of imbalanced nutrition: less than body requirements after noting a weight loss of 30 lb. Which intervention should the nurse add to the plan of care for this patient?

Prescribe a high-calorie, high-protein diet with six small meals a day

The nurse is caring for a patient diagnosed with cor pulmonale. What symptoms assessed by the nurse correlate with the assigned diagnosis?

Presence of edema in the ankles

The nurse understands that which description best characterizes chronic obstructive pulmonary disease (COPD)?

Progressive persistent expiratory airflow limitation

A patient experiences exacerbations of wheezing and breathlessness when exercising in the early morning. The nurse learns that patient has a three-year history of sinusitis. What is appropriate to be included on the patient's plan of care?

Referring the patient to a surgeon for the removal of nasal polyp

A patient who has a family history of α1-protease inhibitor deficiency reports fever of unknown cause, malaise, and cough associated with purulent sputum. Which test should the patient undergo regularly to assess the severity of the condition?

Spirometry

The nurse identifies that which treatment strategy will likely be prescribed for a patient with cor pulmonale, arteriolar vasoconstriction, and a PaO2 of 50 mm Hg?

Supplemental oxygenation

While caring for a patient with a chronic cough and dyspnea associated with the inflammation of lung parenchyma, a nurse instructs the patient, "You should reduce the salt in your diet." What is the reason behind this instruction?

The patient has cor pulmonale

The nurse is caring for a child who is suspected of having cystic fibrosis (CF). The nurse knows that the gold standard of diagnosing this condition is:

The sweat chloride test

A patient is experiencing an exacerbation of chronic obstructive pulmonary disease (COPD) and requires supplemental oxygen. To deliver the precise amount of oxygen, the nurse should use which type of equipment?

Venturi mask

The nurse identifies the nursing diagnosis of activity intolerance for a patient with asthma. In patients with asthma, the nurse assesses for which etiologic factor for this nursing diagnosis?

Work of breathing

Which treatment may increase restlessness and insomnia in a patient with chronic obstructive pulmonary disease (COPD)?

β 2 agonists

The health care provider has prescribed salmeterol for a patient with asthma. In reviewing the use of dry powder inhalers (DPIs) with the patient, what instructions should the nurse provide?

"Close lips tightly around the mouthpiece and breathe in deeply and quickly."

The health care provider has prescribed salmeterol for a patient with asthma. In reviewing the use of dry powder inhalers (DPIs) with the patient, what instructions should the nurse provide? Multiple choice question "Close lips tightly around the mouthpiece and breathe in deeply and quickly." "To administer a DPI, you must use a spacer that holds the medicine so that you can inhale it." "You will know you have correctly used the DPI when you taste or sense the medicine going into your lungs." "Hold the inhaler several inches in front of your mouth and breathe in slowly, holding the medicine as long as possible."

"Close lips tightly around the mouthpiece and breathe in deeply and quickly." The patient should be instructed to tightly close the lips around the mouthpiece and breathe in deeply and quickly to ensure the medicine moves down deeply into the lungs. Dry powder inhalers do not require spacer devices. The patient may not taste or sense the medicine going into the lungs. The inhaler should not be placed several inches in front of the mouth.

Which instruction does the nurse document in the diet plan for the patient with chronic obstructive pulmonary disease (COPD)?

"Eat egg custard every day."

The health care provider has prescribed an inhaled corticosteroid for the patient with asthma. The nurse should provide which instructions to the patient regarding the use of a dry powder inhaler (DPI)?

"Hold your breath for at least 10 seconds to increase medication absorption by your lungs."

The health care provider has prescribed an inhaled corticosteroid for the patient with asthma. The nurse should provide which instructions to the patient regarding the use of a dry powder inhaler (DPI)? Multiple choice question "Shake the canister vigorously before use." "Keep your inhaler in a warm, humid room to prevent crystallization." "Keep your mouth open slightly to increase dispersion of the medication." "Hold your breath for at least 10 seconds to increase medication absorption by your lungs."

"Hold your breath for at least 10 seconds to increase medication absorption by your lungs." The patient should hold the breath for as long as possible to increase the amount of medication absorbed by the lungs. The DPI should not be shaken and should be stored in a cool area, and the patient's mouth should be closed tightly around the mouthpiece of the inhaler.

The registered nurse is teaching a student nurse about care management in a patient with oxygen saturation less than 90 percent, dyspnea, breathlessness, and forced expiratory volume in one second (FEV1) less than 60 percent. Which statement made by the student nurse indicates effective learning?

"I should advise the patient to avoid smoking and occupational exposure to irritants."

The nurse provides education to a caregiver of a patient with bloody sputum, dyspnea, fever, chills, and chest pain. Which statement made by the caregiver indicates effective learning?

"I should ask the patient to drink three liters of water daily."

The nurse teaches a patient regarding the administration of fluticasone through an inhaler. Which statements made by the patient indicate effective learning? Select all that apply. Multiple selection question "I have to shake the medicine bottle before use." "I should keep my medicine away from humid places." "I should engage the lever while loading the medicine." "I should hold my breath for 20 to 30 seconds after a puff." "I have to raise my head forward and breathe into my inhaler."

"I should keep my medicine away from humid places." "I should engage the lever while loading the medicine." The nurse should advise the patient to store the medication in a dry place away from humid places to avoid clumping of the medication. The patient should load the medication into the inhaler and engage the lever to allow the medication to become available. The patient should not shake the medicine bottle. The patient should hold the breath for 10 seconds or as long as possible to disperse the medicine into the lungs. The patient should tilt the head back and breathe out to get maximum air out of lungs, and then inhale the medication. The patient should not breathe into the inhaler, because this will affect the dose.

A patient has frequent attacks of difficulty breathing and wheezing. The nurse is teaching the caregiver about the proper way of positioning the patient to decrease the sense of panic during an attack. Which statement of the caregiver indicates the need for further teaching?

"I should make the patient lay in a side-lying position."

A patient with chronic obstructive pulmonary disease (COPD) has severe dyspnea. The nurse is educating the patient about necessary interventions to conserve energy and avoid further complications. Which statement made by the patient indicates the need for further teaching?

"I should perform deep breathing and effective coughing before meals."

The nurse is evaluating an asthmatic patient's knowledge of self-care. Which statement by the patient indicates an adequate understanding of the instructions given?

"I will keep my rescue inhaler with me at all times."

The nurse is evaluating an asthmatic patient's knowledge of self-care. Which statement by the patient indicates an adequate understanding of the instructions given? Multiple choice question "I will keep my rescue inhaler with me at all times." "I do not need to get a flu shot because I'm under age 50." "I will use my peak flow meter only when I feel like I'm getting sick." "I will use my corticosteroid inhaler only when I feel short of breath."

"I will keep my rescue inhaler with me at all times." Part of a rescue plan for asthma patients is to have access to a short-acting bronchodilator, such as albuterol, to use for rapid control of symptoms. Asthma patients should get a flu shot annually, and the self-monitoring of one's "Personal Best" with a peak flow meter should be done at least daily as part of an asthma action plan. Corticosteroids cannot abort an asthma attack.

The nurse provides home care instructions to a patient who has emphysema. Which statement by the patient indicates correct understanding of the teaching?

"I'll report any change in the color of my sputum."

Which statement made by the patient with chronic obstructive pulmonary disease (COPD) indicates a need for further teaching regarding the use of an ipratropium inhaler?

"If my breathing gets worse, I should keep taking extra puffs of the inhaler until I can breathe more easily."

The nurse is teaching a patient pursed-lip breathing (PLB). What instructions by the nurse are correct?

"Make breathing out (exhalation) three times longer than breathing in (inhalation)."

The nurse determines that the patient understood medication instructions about the use of a spacer device when taking inhaled medications after hearing the patient state what as the primary benefit?

"More of the medication will get down into my lungs to help my breathing."

The nurse determines that the patient understood medication instructions about the use of a spacer device when taking inhaled medications after hearing the patient state what as the primary benefit? Multiple choice question "I will pay less for medication because it will last longer." "More of the medication will get down into my lungs to help my breathing." "Now I will not need to breathe in as deeply when taking the inhaler medications." "This device will make it so much easier and faster to take my inhaled medications."

"More of the medication will get down into my lungs to help my breathing." A spacer assists more medication to reach the lungs, with less being deposited in the mouth and the back of the throat. It does not affect the cost or the increase the speed of using the inhaler.

The nurse reviews pursed-lip breathing with a 61-year-old patient with emphysema. Which statement made by the patient indicates correct understanding of this technique?

"My exhalation should be longer than my inhalation."

The nurse teaches a student nurse regarding the administration of corticosteroids to a patient with wheezing, dyspnea and chest tenderness. Which statement made by the student nurse indicates effective learning? Multiple choice question "Taper the intravenous dose of corticosteroids slowly." "Administer the patient's corticosteroids every 12 hours." "Lower the dose of inhaled corticosteroids to prevent asthma relapse." "Provide inhaled corticosteroids to the patient while still in the hospital."

"Provide inhaled corticosteroids to the patient while still in the hospital. The patient has asthma, a chronic inflammatory disorder of the airways. Corticosteroid inhalation may lead to local infections such as candidiasis and sore throat. Therefore the nurse should provide inhaled corticosteroids to the patient while still in the hospital to check the effectiveness of the therapy and provide immediate relief. Corticosteroids should be administered every four to six hours, and the dose of intravenous corticosteroids should be tapered rapidly. High-dose inhaled corticosteroids help in preventing asthma relapse.

The nurse provides information to a group of nursing students about women with cystic fibrosis (CF) regarding conception and pregnancy. Which statement made by a student nurse related to female patients with CF indicates understanding of the education?

"She may have menstrual irregularities and secondary amenorrhea."

A patient with asthma is prescribed formoterol. What should the nurse include in the medication education provided to the patient? Select all that apply.

"Side effects include cold or flu-like symptoms." "Formoterol is not the only medication that you will need to treat your asthma." "If wheezing gets worse, there are other types of medications that are more beneficial than formoterol."

The nurse is teaching a student nurse about breath sounds in a patient with asthma. Which statement made by the student nurse indicates the need for further teaching? Multiple choice question "The patient wheezes loudly during expiration when the asthma is severe." "The patient has no audible sound from the airway when the asthma is severe." "The patient may wheeze during inspiration and expiration as the asthma progresses." "The patient with asthma must able to move enough air through the airway to produce the sound."

"The patient wheezes loudly during expiration when the asthma is severe." The patient with a minor attack may wheeze loudly during expiration, but the patient experiencing a severe asthma attack does not wheeze; he or she may have no audible sound on auscultation due to decreased airflow into the lungs. The patient must be able to move enough air through the airway to wheeze. Wheezing usually begins with exhalation and may be present during both inspiration and expiration as asthma progresses.

The nurse provides information to a student nurse about antibiotic therapy for a patient with cystic fibrosis (CF) who experiences a productive cough, PaO2 of 50 mmHg, arteriolar vasoconstriction, and cor pulmonale. Which statement made by the student nurse indicates effective learning?

"The patient will require prolonged high-dose antibiotic therapy."

The nurse teaches a group of nursing students about lung volume parameter in patients with asthma. Which statement made by a student nurse indicates the need for further teaching?

"With asthma, there is increased forced expiratory volume in one second."

The nurse teaches a group of nursing students about lung volume parameter in patients with asthma. Which statement made by a student nurse indicates the need for further teaching? Multiple choice question "With asthma, there is increased residual volume." "With asthma, there is increased total lung capacity." "With asthma, there is increased forced expiratory volume in one second." "With asthma, there is a normal forced expiratory volume to forced vital capacity ratio."

"With asthma, there is increased forced expiratory volume in one second." Lung volumes help to determine the reversibility of bronchoconstriction and establish the diagnosis of asthma. The forced expiratory volume in one second decreases in the patient with asthma. The total lung capacity in a patient with asthma increases due to increase in breathing. Therefore the total residual volume increases. The ratio of forced expiratory volume to forced vital capacity is usually normal or decreased in the patient with asthma.

Which nursing instructions would be beneficial to the patient who has shortness of breath, wheezing, and chest tightness? Select all that apply. Multiple selection question "You should avoid contact with furred animals." "You should wash bed covers in hot water and detergent." "You should avoid wearing masks in cold climate conditions." "You should take aspirin when you have shortness of breath." "You should take propanol when you have excess wheezing." "You should ensure that the household does not have any cockroaches."

"You should avoid contact with furred animals." "You should wash bed covers in hot water and detergent." "You should ensure that the household does not have any cockroaches." Shortness of breath, wheezing, and chest tightness indicate that the patient has asthma. Fur acts as an irritant and increases the allergic reactions associated with asthma. Dust mites also trigger asthma, so the patient should wash bed covers with hot water and detergent because this reduces allergens. Danders such as cockroach remains and droppings trigger asthma. Propanol is a nonselective β-blocker that inhibits bronchodilation and should be avoided by patients with asthma. Cold climate conditions are an irritant that triggers asthma, so the patient should wear a mask or scarf in a cold environment. Aspirin precipitates attacks of asthma; therefore the patient should avoid taking aspirin.

The nurse is educating a patient who was recently diagnosed with asthma about the use of a peak flow meter. Which statement is the nurse's priority? Multiple choice question "The peak flow meter should be used weekly." "The peak flow meter reduces asthma attacks." "The peak flow meter should be used before the use of an inhaler." "You should find your personal best peak flow reading for comparison."

"You should find your personal best peak flow reading for comparison." It is important that the patient identify his or her personal best readings to evaluate when the symptoms are evolving into a medical emergency. The peak flow meter does not reduce asthma attacks; it helps monitor symptoms of asthma. It should be used at least twice a day for the first two weeks to determine the patient's personal best, which will be used to monitor airway constriction. The peak flow meter is used after the use of an inhaler, not before, to measure the effectiveness of the medication.

A nurse who is teaching a program about cultural and ethnic health disparities that contribute to obstructive pulmonary diseases would include which statements in the presentation? Select all that apply.

-"Cystic fibrosis is not very common in African Americans or Hispanics." -"Asthma prevalence and age-adjusted death rates are the highest among Puerto Ricans." -"Hispanics have lower death rates from chronic obstructive diseases than other ethnic groups." -"Despite higher rates of smoking in other groups, whites have the highest incidence of chronic obstructive pulmonary diseases."

The nurse teaches a patient regarding the administration of fluticasone through an inhaler. Which statements made by the patient indicate effective learning? Select all that apply.

-"I should keep my medicine away from humid places." -"I should engage the lever while loading the medicine."

The nurse is explaining the pathophysiology of asthma to a patient. Which is the most appropriate explanation?

-"Inflammation causes bronchoconstriction, hyperreactivity, and edema of the airways."

A patient with asthma is prescribed formoterol. What should the nurse include in the medication education provided to the patient? Select all that apply.

-"Side effects include cold or flu-like symptoms." -"Formoterol is not the only medication that you will need to treat your asthma." -"If wheezing gets worse, there are other types of medications that are more beneficial than formoterol."

A patient presents with acute exacerbation of asthma. The nurse expects which strategies will be included in the treatment plan? Select all that apply.

-Administration of 100% oxygen -Intravenous administration of corticosteroids -Nebulization with short-acting β2-adrenergic agonists (SABAs)

A patient with cystic fibrosis is being treated by the nurse with chest physiotherapy for the left upper lobe (LUL) of the lung. Put the steps in the order that they should be performed.

-Albuterol (Proventil) nebulizer therapy -Position patient on right side with head up -Application of percussion and vibration -Effective coughing with expectoration

A patient with chronic obstructive pulmonary disease (COPD) needs to be taught about effective huff coughing in a stepwise manner. In which order should the nurse put the steps for teaching the patient?

-Assume a sitting position -Inhale slowly through your mouth -Hold your breath for 2 to 3 seconds -Forcefully exhale quickly

A patient presents to the emergency department with acute exacerbation of asthma. What actions should the nurse perform to monitor the patient's respiratory and cardiovascular systems? Select all that apply.

-Auscultate the lung sounds -Measure blood pressure and respiratory rate -Monitor arterial blood gases (ABGs) and pulse oximetry

When teaching a patient with asthma about ways to reduce the severity of asthma and asthma attacks, which measures should be included? Select all that apply.

-Avoid food irritants -Avoid animals with fur -Identify personal triggers

The nurse is assessing the respiratory system of an 87-year-old patient admitted with emphysema. Which assessment findings does the nurse expect? Select all that apply.

-Barrel chest -End-expiratory wheezes

A patient with an acute attack of asthma is in a state of panic. Which nursing measures help to relieve the panic? Select all that apply.

-Be calm, quiet, and reassuring. -Encourage pursed-lip breathing -Utilize the "talking down" technique

A patient is prescribed albuterol with a metered dose inhaler (MDI). After taking the cap off and shaking the inhaler, the nurse should instruct the patient to perform the steps of taking the medication in what order?

-Breathe out completely -Hold the inhaler straight -Breathe slowly through mouth -Press down the inhaler -Hold the breath for 10 seconds -Wait for one minute after every puff

A patient who has bronchiectasis asks the nurse, "What conditions would warrant a call to the clinic?" Which of these would the nurse include in the answer? Select all that apply.

-Chest pain -Fever and chills -Blood clots in the sputum -Increased sputum production

A patient with chronic obstructive pulmonary disease (COPD) is receiving oxygen therapy through a mask. Which nursing actions should the nurse perform to ensure proper care of the patient? Select all that apply.

-Choose the optimal oxygen device -Assess the need to adjust the oxygen flow rate -Monitor for signs of adverse effects of oxygen therapy

The nurse is teaching a class about smoking cessation. Select the respiratory-related symptoms associated with cigarette smoking. Select all that apply.

-Chronic cough -Decreased sense of taste -Paralysis of the cilia inside the lungs

Which should the nurse include when providing information related to health promotion and disease prevention to a patient with cystic fibrosis (CF)? Select all that apply.

-Consume more fluids and caloric supplements -Learn about central venous access devices (CVAD) that may be used for long-term medication administration

What are late complications of cystic fibrosis (CF) caused by pulmonary hypertension? Select all that apply.

-Cor pulmonale -Respiratory failure

A 61-year-old patient with asthma is admitted to the hospital. The nurse understands that symptoms of asthma include what? Select all that apply.

-Cough -Wheezing -Chest tightness

What adventitious sounds may be auscultated when assessing a patient with pleuritic chest pain and dyspnea? Select all that apply.

-Crackles -Rhonchi -Wheezes

A patient with a blood sugar level of 210 mg/dL has bone density of -1.5 standard deviation. The nurse expects what diagnostic findings? Select all that apply.

-Destruction of islet cells in the pancreas -Insufficient levels of testosterone hormone -Chronically elevated inflammatory cytokines

A patient with advanced stage chronic obstructive pulmonary disease (COPD) experiences significant weight loss. The nurse expects that what will be included on the patient's plan of care? Select all that apply.

-Encourage rest before meals -Encourage high-protein supplements between meals

A patient is using a peak flow meter to check asthma. What is the order of interventions that the patient should follow while using the device?

-Ensure that the indicator is at the bottom of the numbered scale -Stand up -Take a deep breath to fill the lungs -Place the mouthpiece in the mouth and close the lips -Blow hard and fast in a single blow

The nurse is planning care for a patient with cystic fibrosis (CF). Which actions by the nurse are most appropriate? Select all that apply.

-Evaluates blood glucose levels -Provides fat-soluble vitamin supplementation -Prepares to provide pancreatic enzyme replacement medications

The nurse is providing education about the risk factors of asthma. Which factors does the nurse explain are associated with asthma? Select all that apply

-Immune response -Genetic predisposition -Exposure to air pollutants

The nurse is providing education about the risk factors of asthma. Which factors does the nurse explain are associated with asthma? Select all that apply.

-Immune response -Genetic predisposition -Exposure to air pollutants

The nurse provides education to a patient who is prescribed a metered-dose inhaler. Which actions taken by the patient indicate the need for further teaching? Select all that apply.

-Inhales more than one puff with each inspiration -Does not shake the metered-dose inhaler before use

What are the advantages of a diskus over a metered-dose inhaler? Select all that apply.

-It requires less manual dexterity. -It contains an easily visible color that indicates the number of doses left

A patient has hyperventilation, dyspnea, nasal flaring, anxiety, and pursed-lip breathing. The nurse concludes that the patient has an ineffective breathing pattern due to alveolar hypoventilation. Which interventions should the nurse implement for this patient to promote safe and effective care? Select all that apply.

-Monitor for respiratory muscle fatigue -Encourage slow, deep breathing and coughing -Elevate the head of the bed and provide an overbed table for the patient

The nurse is caring for a patient with chronic obstructive pulmonary disease (COPD). What does the nurse recognize are the predominant inflammatory cells in this disorder? Select all that apply.

-Neutrophils -Lymphocytes -Macrophages

Which complications in a patient with chronic obstructive pulmonary disorder (COPD) require acute intervention? Select all that apply.

-Pneumonia -Cor pulmonale -Exacerbations

Which diagnostic findings indicate that a patient has asthma? Select all that apply.

-Positive skin test for allergens -Serum and sputum eosinophilia -Chest x-ray indicating hyperinflation

The nurse expects what assessment findings in a patient with cystic fibrosis and bronchiectasis? Select all that apply.

-Protuberant abdomen -Emaciated appearance of the extremities

When performing pilocarpine iontophoresis in a patient with steatorrhea, the nurse should perform the steps of the procedure in what order?

-Provide education to the patient -Place pilocarpine on the skin -Wait for five minutes -Collect sweat onto a gauge -Analyze the level of chloride concentration

The nurse is teaching a patient how to use an inhaler. What steps should the nurse teach the patient? Please place in sequential order.

-Shake the inhaler -Breathe out all the way -Place the inhaler near or in your mouth as instructed by your health care provider (HCP). -As you start breathing in slowly through the mouth, press down on the inhaler one time -Keep breathing in slowly and as deeply as possible -Hold your breath as you count to 10 slowly

What are complications associated with pancreatic insufficiency? Select all that apply

-Steatorrhea -Low body mass index

What are complications associated with pancreatic insufficiency? Select all that apply.

-Steatorrhea -Low body mass index

A nurse observes a patient with asthma using a metered-dose inhaler. The nurse should correct which patient actions? Select all that apply.

-The patient holds the breath for five seconds -The patient presses the meter dose inhaler gently

The nurse is caring for a patient with chronic obstructive pulmonary disease. About what medications will the nurse educate the patient that have the side effects of a feeling of early satiety and a bloating sensation? Select all that apply.

-Theophylline -Corticosteroids

A smoker with forced expiratory volume in one second (FEV1) of 50 percent and arterial blood oxygen saturation less than 85 percent is referred for pulmonary rehabilitation (PR). Which interventions does the nurse anticipate will be part of the PR? Select all that apply.

-Using the internet to deliver daily motivational messages -Teaching exercises that focus on the muscles used for ambulation along with upper limb exercises

The patient is prescribed a high-flow oxygen delivery system. Which of these are high-flow devices? Select all that apply.

-Venturi mask -Mechanical ventilator

What discharge instructions should the nurse include for the patient with asthma? Select all that apply.

-Wash the nebulizer regularly -Medication noncompliance may lead to exacerbations -It is important to know the purpose and side effects of prescribed medications.

Which complications, along with edema in the ankles, does the nurse expect in a patient with chronic obstructive pulmonary disease (COPD)? Select all that apply.

-Weight gain -Hepatomegaly -Jugular vein distension

The patient has a prescription to receive 600 mg guaifenesin every four hours as needed (PRN). Available are 400 mg tablets. How many tablets should the nurse prepare to give?

1.5 tablets

The nurse is overseeing an exercise program for patients with mild chronic obstructive pulmonary disease (COPD). Part of the program involves walking. Vital signs are taken after walking. The nurse becomes concerned when a 60-year-old patient's pulse rate is what?

120 beats/minute

The patient has a prescription for albuterol 5 mg via nebulizer. Available is a solution containing 2 mg/mL. Calculate how many mL the nurse should use to prepare the patient's dose. Fill in the blank using one decimal place. __ mL

2.5

The patient has a prescription to receive methylprednisolone sodium succinate 150 mg intravenous (IV) push stat. Available is a solution containing 60 mg/mL. How many mL of methylprednisolone should the nurse administer?

2.5 mL

A patient with a tracheostomy has difficulty breathing. Which device will be the most feasible for the patient to breathe through the tracheostomy tube?

4

After assessing the peak flow meter readings of a patient with asthma, the nurse concludes that the patient should inhale quick relief medicine right away. What could be the peak flow number of the patient?

40 percent of personal best number

After assessing the peak flow meter readings of a patient with asthma, the nurse concludes that the patient should inhale quick relief medicine right away. What could be the peak flow number of the patient? Multiple choice question 40 percent of personal best number 60 percent of personal best number 75 percent of personal best number 85 percent of personal best number

40 percent of personal best number The patient with asthma whose peak flow meter reading is in the red zone or less than 50 percent of his or her personal best number is instructed to inhale a quick relief medicine immediately, which is a short-acting β-adrenergic agonist. Therefore the peak flow meter reading of the patient could be 40 percent of his or her personal best number. The patient who has a peak flow meter reading between 50 percent and 80 percent will require an increase in medication dosage, but not immediate treatment. Therefore the patient with a peak flow meter reading of 60 percent or 75 percent of his or her personal best number will not require immediate treatment. The patient with a peak flow meter reading of 85 percent of his or her personal best number is in the green zone of peak flow zone and does require any further change in the treatment strategy.

An asthmatic patient has been prescribed 4 puffs of salmeterol daily. The canister of the meter dose inhaler has 200 puffs. The nurse instructs the patient that the canister will be empty and will need to be replaced in how many days? Record your answer using a whole number. __ days

50

A patient is prescribed an inhaler for treatment of asthma. The medication canister contains 200 puffs, scheduled to last for 25 days. The nurse should instruct the patient to inhale how many puffs per day to receive the prescribed amount? Record your answer using a whole number. ________

8

The nurse is caring for a five-year-old child with low body weight and poor growth. The parent reports that the child is unable to digest fat- and protein-rich foods. The nurse suspects that the child has what condition?

A child with low body weight, poor growth, and malabsorption of fat and protein may have pancreatic insufficiency, which can lead to atrophy and cyst formation. This may result in pancreatic dysfunction, which would affect synthesis of enzymes such as lipase, amylase, and protease, which help digest fats and proteins. Ultimately, this affects the growth and weight of the child. Liver cirrhosis occurs due to mucus deposition in the liver. Chronic bronchiolitis occurs due to mucus accumulation in the small airway. Cystic fibrosis-related diabetes mellitus (CFRD) occurs due to destruction of islet cells in the pancreas.

Which test result identifies that a patient with asthma is responding to treatment?

A decrease in exhaled nitric oxide

Which test result identifies that a patient with asthma is responding to treatment? Multiple choice question An increase in CO 2 levels A decrease in exhaled nitric oxide A decrease in white blood cell count An increase in serum bicarbonate levels

A decrease in exhaled nitric oxide Nitric oxide levels are increased in the breath of people with asthma. A decrease in the exhaled nitric oxide concentration suggests that the treatment may be decreasing the lung inflammation associated with asthma and adherence to treatment. An increase in CO 2 levels, decreased white blood cell count, and increased serum bicarbonate levels do not indicate a positive response to treatment in the asthma patient.

The nurse is caring for the patient with chronic obstructive pulmonary disease who is undernourished and underweight. Which steps can the nurse take to improve the patient's nutritional status? Select all that apply.

A diet high in calories and protein, moderate in carbohydrates, and moderate to high in fat is recommended and can be divided into five or six small meals a day. High-protein, high-calorie nutritional supplements can be offered between meals. Nonprotein calories should be divided evenly between fat and carbohydrate, but avoid overfeeding the patient. Fluid intake should be at least 3 L/day unless contraindicated by other medical conditions. Fluids should be taken between meals (rather than with them) to prevent excess stomach distention and to decrease pressure on the diaphragm.

The nurse expects that what diagnosis will be made for a patient that has permanent, abnormal dilation of medium-sized bronchi, inflammation, and elastic and muscular structural destruction of the bronchial wall?

A patient who has permanent, abnormal dilation of medium-sized bronchi, inflammation, and elastic and muscular structural destruction of the bronchial wall has bronchiectasis. Cystic fibrosis (CF) is an autosomal recessive, multi-system disease, which alters transport of sodium and chloride ions in and out of epithelial cells. Cor pulmonale occurs due to an increase in vascular resistance or pulmonary hypertension. Pneumothorax is a condition where air accumulates in the plural space.

A patient with cystic fibrosis (CF) is hospitalized with exacerbation of symptoms, wheezing, purulent sputum, and a fasting blood glucose level of 194 mmol/L. The nurse anticipates that what medication will be prescribed?

A patient with a blood glucose level of 194 mmol/L, purulent sputum, and wheezing associated with destruction of islets of pancreas has cystic fibrosis-related hyperglycemia (CFRD). The patient with CFRD has characteristics of both type 1 and type 2 diabetes mellitus. The patient should be administered insulin. Ivacaftor is useful in the patient with a G551D mutation of CF. Dornase alfa degrades deoxyribonucleic acid (DNA) of neutrophils in the sputum of the patient with CF. Pancrelipase helps to manage pancreatic insufficiency.

A male patient experiences a cough, perfuse sweating, local hypoxia, and arteriolar vasoconstriction. The patient states, "I hope that this condition allows me to have biologic children." What is the best nursing response?

A patient with a cough, profuse sweating, local hypoxia, and arteriolar vasoconstriction has cystic fibrosis. Men with cystic fibrosis may fail to develop vas deferens in utero. Therefore the sperm remain in the testes and do not pass through the penile urethra. The nurse may suggest assisted reproductive technology to the patient. The nurse should not depend upon a case manager or the primary health care provider to teach the patient. Patients with CF have normal production of sperm in spite of having testosterone insufficiency.

What is the purpose of dornase alfa, which is often prescribed for a patient with cystic fibrosis (CF)?

A patient with a cough, purulent mucus, local hypoxia, and arteriolar vasoconstriction has cystic fibrosis (CF). Management of CF aims to relieve airway obstruction and to control infection. Dornase alfa helps to degrade deoxyribonucleic acid in the sputum, thereby increasing the airflow and reducing the number of acute pulmonary exacerbations. Beta-adrenergic agonists help to avoid bronchospasm. Pancrelipase acts as a supplement for lipase, amylase and protease to manage pancreatic insufficiency. Chest tube drainage helps to control episodes of pneumothorax.

The nurse provides information to a student nurse about antibiotic therapy for a patient with cystic fibrosis (CF) who experiences a productive cough, PaO2 of 50 mmHg, arteriolar vasoconstriction, and cor pulmonale. Which statement made by the student nurse indicates effective learning?

A patient with a cough, sputum, PaO2 of 50 mm Hg, arteriolar vasoconstriction, and cor pulmonale has CF-related infection. Most patients with CF are at a risk of lung infection. Hence a standard treatment of prolonged antibiotic therapy helps to treat exacerbations and includes chronic suppression therapy in conjunction with airway clearance. The patient must be supported with two intravenous antibiotics with different mechanisms of action. Two to four weeks of intravenous antibiotic therapy may benefit the patient with severe exacerbations. For mild disease exacerbations, oral (versus intravenous) antibiotics will be prescribed.

A patient has hyperventilation, dyspnea, nasal flaring, anxiety, and pursed-lip breathing. The nurse concludes that the patient has an ineffective breathing pattern due to alveolar hypoventilation. Which interventions should the nurse implement for this patient to promote safe and effective care? Select all that apply.

A patient with an ineffective breathing pattern has alveolar hypoventilation. The nurse must monitor the patient for respiratory muscle fatigue to determine the need for ventilation. The nurse should encourage the patient to breathe slowly and deeply and to cough, mobilizing pulmonary secretions for effective airway clearance. The nurse should elevate the head of the patient's bed and provide an overbed table to reduce respiratory efforts. Bronchodilators should be administered because they help increase the gas exchange. The nurse should provide factual information concerning diagnosis, treatment, and prognosis to a patient with anxiety-related breathlessness.

The nurse provides education to a caregiver of a patient with bloody sputum, dyspnea, fever, chills, and chest pain. Which statement made by the caregiver indicates effective learning?

A patient with bloody sputum, dyspnea, fever, chills, and chest pain has bronchiectasis. The focus of the care is to promote drainage and removal of mucus in the airway. The patient must hydrate by drinking three liters of water per day. This helps to liquefy the secretions and thereby make it easier to remove them. The patient should drink low-sodium fluids to avoid systemic fluid retention. The patient will be anorexic; it is difficult to administer high-calorie foods to this patient. Oral hygiene, such as cleaning the patient's mouth and mucus crusts, promotes oral health, prevents infection, and may help increase appetite. If bleeding is suspected, the patient must be placed in a side-lying position with the suspected bleeding side down.

Which nursing intervention will the nurse include in the teaching plan for a patient with difficulty in breathing due to low oxygen level in the blood?

A patient with chronic obstructive pulmonary disorder (COPD) has dyspnea and hypoxemia due to poor ventilation. For males, erectile dysfunction can occur with COPD, as with many chronic diseases. The nurse may suggest that the patient refrain from sexual activity after eating to prevent breathlessness during intercourse. The patient should use pursed lips to breathe slowly. It is better to plan sexual activity during the day, when the patient's breathing is best. The patient should choose a less stressful position during intercourse and avoid the missionary position.

The nurse identifies that which treatment strategy will likely be prescribed for a patient with cor pulmonale, arteriolar vasoconstriction, and a PaO2 of 50 mm Hg?

A patient with cor pulmonale, arteriolar vasoconstriction, and partial oxygen of 50 mm Hg (hypoxemia) is in respiratory failure. The patient needs supplementation of oxygen for speedy recovery. The patient with a large pneumothorax requires chest tube drainage. The patient with chronic cystic fibrosis requires lung transplantation. The patient with hemoptysis requires bronchial artery embolization.

A patient with cystic fibrosis (CF) experiences oily, irregular bowel movements and has a fat-soluble vitamin deficiency. The nurse anticipates what prescription?

A patient with cystic fibrosis (CF) who has oily, irregular bowel movements indicates steatorrhea in the patient with pancreatic insufficiency. This occurs due to malabsorption of fats and fat-soluble vitamins such as A, D, K, and E. Hence the patient should take pancrelipase before each meal. Pancrelipase helps to manage pancreatic insufficiency, because it includes pancreatic enzyme replacement for lipase, protease, and amylase. Azithromycin and tobramycin help to reduce exacerbations of CF due to Pseudomonas. Antimicrobial administration helps to control severe exacerbations of CF

Which intervention by the nurse would be most appropriate for safe and effective care for a patient with expiratory airflow obstruction, ineffective cough, decreased airway humidity, and abnormal breath sounds?

A patient with ineffective cough, decreased airway humidity, and abnormal breath sounds has ineffective airway clearance. Hence the nurse must instruct the patient to inhale deeply by bending slightly forward and perform three to four huffs against an open glottis to avoid airway collapse upon exhalation. The nurse should monitor for respiratory muscle fatigue to determine the need for respiratory ventilation. The nurse should set up and administer oxygen to a patient with impaired gas exchange. Auscultating breath sounds to assess the ventilation and the presence of adventitious sounds helps obtain the breathing patterns of a patient who is on treatment.

Which cells cause inflammation in a patient with an inflamed airway and productive cough who has Pseudomonas aeruginosa infection?

A patient with inflammation of the airway, productive cough, and Pseudomonas aeruginosa infection may have cystic fibrosis. The chronic inflammation may lead to infection. Neutrophils release inflammatory mediators like interleukins, oxidants, and proteases that contribute to the progression of the disease. Mast cells, lymphocytes, and macrophages are the inflammatory cells that mediate inflammation of the airways in patients with asthma and chronic pulmonary disease

Which condition may develop if a patient with localized hypoxia and arteriolar vasoconstriction who has blebs and large cysts in the lungs is left untreated?

A patient with localized hypoxia and arteriolar vasoconstriction has lung disorder with changes in the bronchial wall. Blebs and large cysts in the lungs are severe manifestations of lung destruction. If the patient is left untreated, the patient may develop pneumothorax. Atelectasis is a complication of asthma due to hyperinflation of the lungs and increased dead space. Hemoptysis is a lung complication that occurs due to erosion of enlarged pulmonary arteries. Chronic bronchitis may be the initial stage of lung disorders.

The registered nurse is teaching a student nurse about care management in a patient with oxygen saturation less than 90 percent, dyspnea, breathlessness, and forced expiratory volume in one second (FEV1) less than 60 percent. Which statement made by the student nurse indicates effective learning?

A patient with oxygen saturation less than 90%, dyspnea, and breathlessness has chronic obstructive pulmonary disease (COPD). The nurse should advise the patient to cease smoking and to avoid exposure to irritants to maintain healthy lungs. The nurse should counsel the patient to stop smoking because it is the only way to slow the progression of COPD. The patient should avoid gas-forming foods such as cabbage, beans, and cauliflower. The nurse should advise the patient to breathe slowly while performing effective Huff coughing. The patient with COPD has to eat more high-calorie food, divided into six small meals per day

What are complications associated with pancreatic insufficiency? Select all that apply

A patient with pancreatic insufficiency may have protein and fat malabsorption. Hence the patient may be thin with frequent, bulky, foul-smelling stools (steatorrhea) and a low body mass index (BMI). Osteopenia occurs in a patient with chromosome 7 mutation's effect on development of bone. Chronic elevation of liver enzymes leads to liver cirrhosis in the patient with cystic fibrosis. Distal intestinal obstruction syndrome results from intermittent obstruction in the terminal ileum.

The nurse recognizes that which intervention will be beneficial to the patient with pleuritic chest pain, dyspnea, wheezing, and mouth bleeding?

A patient with pleuritic chest pain, dyspnea, wheezing, and bleeding may have bronchiectasis with hemoptysis. Bronchiectasis is difficult to treat. The nurse should aim to treat acute flare-ups and should try to prevent a decline in lung function. Therefore the nurse should teach the patient about the benefits of pneumococcal and influenza vaccination. The nurse should allow the patient to consume dietary liquids after clearing the mouth of dried sputum crusts. This helps to increase the appetite of the patient. The nurse should raise the head of the bed and place the patient in a side-lying position when mouth bleeding occurs, to allow expectoration and to prevent aspiration. Hydrating the respiratory system with hypertonic saline improves expectoration of secretions and is likely to be beneficial.

The nurse is teaching energy conservation techniques to a patient with severe dyspnea and oxygen saturation of 50 mm Hg. Which action of the patient indicates effective learning?

A patient with severe dyspnea and oxygen saturation of 50 mm Hg has chronic obstructive pulmonary disease (COPD). The patient must follow measures to conserve energy during daily living. Assuming a tripod posture and placing the mirror on the table to use an electric razor conserves much more energy than standing in front of a mirror to shave. The patient should try to sit as much as possible when performing activities. The patient should use the upper thoracic and neck muscles to breathe rather than the diaphragm. This will help the patient overcome difficulty performing activities using the upper limbs, and thus reduces dyspnea. The patient must exhale while pulling, pushing, or lifting and inhale while at rest.

The nurse is caring for a patient with an oxygen saturation of 45 percent of personal best who stops breathing while sleeping and has a tendency to sleep during the day. Which intervention by the nurse will help the patient most while recovering?

A patient with sleep apnea has oxygen saturation of 45 percent, stops breathing while sleeping, and has a tendency to sleep during the day. The patient with severe breathlessness and hypoxemia may need immediate medical attention and regular follow-up. Therefore the patient should be encouraged to use typed messages such as texting and instant messages to communicate. The patient should avoid exercise or walking during the attack because severe dyspnea may lead to respiratory failure. The patient should have an adequate diet to prevent weight loss. Hence, the patient should avoid consuming more fluids at mealtime. Patients with severe sleep apnea and dyspnea cannot speak in sentences because of difficulty in breathing.

The licensed practical nurse is caring for a chronic obstructive pulmonary disease (COPD) patient who has severe hypoxemia. The primary health care provider prescribed oxygen administration for this patient. Which intervention can the nurse perform for the patient?

Adjust the oxygen flow rate depending on the desired oxygen level

The patient is receiving 3 L of oxygen (O 2) via nasal cannula. Which action by the nurse is most appropriate? Select all that apply. Multiple selection question Assesses eyes for dryness Realizes that humidification is never needed Adjusts humidification according to patient comfort Assesses the bubble-through humidifier if humidity is used Assures that the patient is wearing the nasal cannula correctly

Adjusts humidification according to patient comfort Assesses the bubble-through humidifier if humidity is used Assures that the patient is wearing the nasal cannula correctly Assessing the bubble-through humidifier if humidity is used, assuring that the patient is wearing the nasal cannula correctly, and adjusting humidification according to patient comfort are correct because oxygen (O 2) obtained from cylinders or wall systems is dry. Dry O 2 has an irritating effect on mucous membranes and dries secretions. A common device used for humidification when the patient has a cannula or a mask is a bubble-through humidifier. It is important for the nurse to assess the bubble-through humidifier if humidity is used to make sure the humidification is on. This adds to the comfort of the patient. The nurse assesses the patient to make sure the nasal cannula is worn correctly for optimal effect. The cannula can become easily dislodged. Humidification is adjusted according to the patient's comfort level. When oxygen levels are 1 to 4 L, the use of humidification may not be the preference of all patients. Believing that humidification is never needed is incorrect because the use of humidification is a patient preference. The nurse should assess the patient's nose for dryness, not the eyes.

The nurse is caring for a patient who is being mechanically ventilated that has a PaCO2 of 60 mm Hg, a respiratory pH of 6.8, and a PaO2 of 60 mm Hg. Which intervention will benefit the patient?

Administering sodium bicarbonate

The nurse is caring for a patient who is being mechanically ventilated that has a PaCO 2 of 60 mm Hg, a respiratory pH of 6.8, and a PaO 2 of 60 mm Hg. Which intervention will benefit the patient? Multiple choice question Administering albuterol Performing a bronchoscopy Providing chest physiotherapy Administering sodium bicarbonate

Administering sodium bicarbonate A patient with PaCO 2 of 60 mm Hg and respiratory pH of 6.8 has severe respiratory acidosis. Therefore such a patient must take sodium bicarbonate to treat extreme acidosis. Bronchodilation by a beta-agonist like albuterol is not possible in the patient with extreme acidosis. Bronchoscopy helps to relieve acute attacks by removing thick mucus plugs. Chest physiotherapy is generally not recommended for asthma because it is too stressful for breathless patients.

An adolescent who has a history of asthma experiences wheezing after vigorous exercise. What nursing instruction is helpful for this patient?

Advise the patient to avoid dry air

An adolescent who has a history of asthma experiences wheezing after vigorous exercise. What nursing instruction is helpful for this patient? Multiple choice question Advise the patient to avoid dry air. Encourage the patient to get exposure to cold air. Recommend the patient to continue vigorous exercise. Advise the patient to avoid swimming in indoor heated pools.

Advise the patient to avoid dry air. The patient is suffering from exercise-induced asthma. The nurse should encourage the patient to avoid exposure to dry air, because it precipitates exercise-induced asthma. Exposure to cold air also precipitates asthma. The patient should be discouraged from performing vigorous exercise, because it can precipitate asthma. Swimming in indoor heated pools should be encouraged over swimming outdoors, because outdoor swimming can trigger asthma attacks.

What would be the appropriate nursing intervention for the patient whose laboratory report shows oxygen saturation of 45 mm Hg and carbon dioxide saturation of 60 mm Hg?

Advise the patient to plan sexual activity during the early afternoon

Which nursing intervention will the nurse include in the teaching plan for a patient with difficulty in breathing due to low oxygen level in the blood?

Advise the patient to refrain from sexual activity after eating

The nurse is caring for the patient with chronic obstructive pulmonary disease (COPD). The nurse will include in the patient plan of care:

Advise the patient to rest at least 30 minutes before eating

What instruction should the nurse give to a patient with asthma who has received a prescription for albuterol and ipratropium nebulization?

After the treatment, cough effectively

The patient has a prescription for each of the inhalers. Which one should the nurse offer to the patient at the onset of an asthma attack?

Albuterol

Which inhaler should the nurse be prepared to administer to the patient at the onset of an asthma attack?

Albuterol

The patient has a prescription for each of the inhalers. Which one should the nurse offer to the patient at the onset of an asthma attack? Multiple choice question Albuterol Salmeterol Beclomethasone Ipratropium bromide

Albuterol Albuterol is a short-acting bronchodilator that should be given initially when the patient experiences an asthma attack. Salmeterol is a long-acting β 2-adrenergic agonist, which is not used for acute asthma attacks. Beclomethasone is a corticosteroid inhaler and is not recommended for an acute asthma attack. Ipratropium bromide is an anticholinergic agent that is less effective than β 2-adrenergic agonists. It may be used in an emergency with a patient unable to tolerate short-acting β 2-adrenergic agonists (SABAs).

The patient has a prescription to use albuterol and beclomethasone inhalers, two puffs each. The nurse determines that the patient needs additional teaching on how to safely self-administer these medications after noting that the patient performs which action?

Albuterol, a β2-adrenergic agonist medication, should be used first to dilate the airways before administration of the corticosteroid beclomethasone. Administering the beclomethasone on a set schedule to prevent an asthma attack and rinsing the mouth following use of the inhalers are correct actions.

A patient is having an asthma attack, and is short of breath and appears frightened. The nurse understands that possible triggers for asthma exacerbations include which factors? Select all that apply. Multiple selection question Alcohol Perfumes Animal dander Humid weather Gastroesophageal reflux disease (GERD)

Alcohol Perfumes Animal dander Gastroesophageal reflux disease (GERD) Alcohol, GERD, animal dander, perfumes, and cold weather (not humid) are all possible triggers for acute asthma exacerbations

A patient develops vascular congestion, edema, and breathlessness within 30 minutes of the consumption of food containing tartrazine. The nurse identifies that which treatment may exacerbate the patient's condition? 1 Inhalation of anticholinergic drugs 2 Intubation through assisted ventilation 3 Administration of intravenous corticosteroid 4 Inhalation of oxygen by non-rebreather mask

Answer3

A patient is experiencing an exacerbation of chronic obstructive pulmonary disease (COPD) and requires supplemental oxygen. To deliver the precise amount of oxygen, the nurse should use which type of equipment? 1 Venturi mask 2 Simple face mask 3 Non-rebreather mask 4 Laryngeal mask airway

Answer: 1

A patient who has a family history of α1-protease inhibitor deficiency reports fever of unknown cause, malaise, and cough associated with purulent sputum. Which test should the patient undergo regularly to assess the severity of the condition? 1 Spirometry 2 Liver enzyme tests 3 Renal hormone tests 4 Computerized tomography

Answer: 1

The nurse is teaching a student nurse about breath sounds in a patient with asthma. Which statement made by the student nurse indicates the need for further teaching? 1 "The patient wheezes loudly during expiration when the asthma is severe." 2 "The patient has no audible sound from the airway when the asthma is severe." 3 "The patient may wheeze during inspiration and expiration as the asthma progresses." 4 "The patient with asthma must able to move enough air through the airway to produce the sound.

Answer: 1

The nurse provides education to a patient with asthma about how to take medication through a metered dose inhaler. Which action performed by the patient indicates effective learning? 1 Cleans the device with water 2 Inhales two puffs, two seconds apart 3 Does not shake the medication before taking it 4 Breathes in rapidly while taking the medication

Answer: 1

What instruction should the nurse give to a patient with asthma who has received a prescription for albuterol and ipratropium nebulization? 1 After the treatment, cough effectively. 2 During the treatment, breathe rapidly through the mouth. 3 For the treatment, it is recommended to be in the side-lying position. 4 Between treatments, it is sufficient to clean the mouth by gargling with water.

Answer: 1

Which finding indicates to the nurse that a patient's respiratory status is improving following an acute asthma exacerbation? 1 Audible wheezing 2 Pursed lip breathing 3 Use of intercostal muscles 4 Oxygen saturation 89% of room ai

Answer: 1

Which treatment may increase restlessness and insomnia in a patient with chronic obstructive pulmonary disease (COPD)? 1 β 2 agonists 2 Anticholinergics 3 Massage and postural drainage techniques 4 Oxygen supplementation through a nasal mask

Answer: 1

The nurse is providing education about the risk factors of asthma. Which factors does the nurse explain are associated with asthma? Select all that apply. 1 Immune response 2 Sedentary lifestyle 3 History of pancreatitis 4 Genetic predisposition 5 Exposure to air pollutants

Answer: 1, 4, 5

What discharge instructions should the nurse include for the patient with asthma? Select all that apply. 1 Wash the nebulizer regularly. 2 Wash the mouth after taking albuterol. 3 Medication noncompliance may lead to exacerbations. 4 It is important to know the purpose and side effects of prescribed medications. 5 Wash respiratory equipment with a solution of one-part water to two parts white vinegar

Answer: 1,3,4

What are the advantages of a diskus over a metered-dose inhaler? Select all that apply. 1 It requires less manual dexterity. 2 It includes coordination of device puffs with inhalation. 3 The medication does not clump upon exposure to the humidity. 4 It contains an easily visible color that indicates the number of doses left. 5 A patient with low forced expiratory volume in one second can easily inhale the medication using the diskus

Answer: 1,4

A smoker with forced expiratory volume in one second (FEV1) of 50 percent and arterial blood oxygen saturation less than 85 percent is referred for pulmonary rehabilitation (PR). Which interventions does the nurse anticipate will be part of the PR? Select all that apply

Answer: 1,5

A patient with asthma experiences anaphylaxis. Which medication should the nurse prepare to administer? 1 Timolol 2 Epinephrine 3 Magnesium sulfate 4 Sodium bicarbonate

Answer: 2

After the inhalation of puffs of mometasone, a patient develops oropharyngeal candidiasis, hoarseness and dry cough. What action should the nurse take to reduce the symptoms? 1 Recommend that the patient pauses between the puffs 2 Instruct the patient to rinse the mouth with water after inhalation 3 Assist the patient in obtaining a spacer or holding device for inhalation 4 Wait until the cough subsides before administering the patient's next dose

Answer: 2

The nurse is explaining the pathophysiology of asthma to a patient. Which is the most appropriate explanation? 1 "An acid-base imbalance causes bronchoconstriction and edema of the airways." 2 "Inflammation causes bronchoconstriction, hyperreactivity, and edema of the airways." 3 "Inflammation causes bronchodilation, hyperreactivity, and pressure of the airways." 4 "An immune response causes bronchodilation, hyperreactivity, and edema of the airways.

Answer: 2

The registered nurse is teaching a student nurse about care management in a patient with oxygen saturation less than 90 percent, dyspnea, breathlessness, and forced expiratory volume in one second (FEV1) less than 60 percent. Which statement made by the student nurse indicates effective learning? 1 "I should advise the patient to eat more cabbage, beans, and cauliflower." 2 "I should advise the patient to avoid smoking and occupational exposure to irritants." 3 "I should advise the patient to breathe rapidly while performing effective Huff coughing." 4 "I should advise the patient to avoid high-calorie foods like butter, cheese, and margarine."

Answer: 2

What would be the appropriate nursing intervention for the patient whose laboratory report shows oxygen saturation of 45 mm Hg and carbon dioxide saturation of 60 mm Hg? 1 Advise the patient to move to a higher-altitude location. 2 Advise the patient to plan sexual activity during the early afternoon. 3 Advise the patient to walk for 30 minutes daily, breathing through the nose. 4 Administer β-adrenergic agonists, as prescribed, immediately after exercise.

Answer: 2

Which assessment finding does the nurse expect when caring for a patient with asthma? 1 pH of 5.11 2 PaCO2 of 30 mm Hg 3 Blood pressure of 110/60 mm Hg 4 Respiratory rate of 25 breaths/minute

Answer: 2

Which intervention is beneficial to a patient with chronic obstructive pulmonary disease (COPD)? 1 Avoiding cold foods 2 Limiting fluids during mealtimes 3 Avoiding frequent meals and snacks 4 Performing physical activity before meals

Answer: 2

A patient has had chronic obstructive pulmonary disease (COPD) for three months. Further assessment reveals that forced expiratory volume in one second (FEV1) is 65 percent. Which treatment option would be most appropriate for this patient? 1 Treatment with salmeterol 2 Treatment with budesonide 3 Treatment with albuterol and ipratropium combination 4 Treatment with budesonide and formoterol combination

Answer: 3

A patient in the emergency ward is dyspneic and speaks unclear words. Assessment findings include a respiratory rate of 45 breaths/minute, a pulse of 130 beats/minute, oxygen saturation of 90%, and neck vein distention. Which treatment does the nurse expect will help alleviate the patient's symptoms? 1 Administration of ipratropium orally 2 Three puffs of albuterol every 30 minutes 3 Supplementary oxygen through nasal cannula 4 Obtaining peak flow rate and monitoring the patient continuously

Answer: 3

A patient with asthma has a body temperature of 102° F and produces purulent sputum. The nurse anticipates that which drug will be prescribed? 1 A sedative 2 A mucolytic 3 An antibiotic 4 Epinephrine

Answer: 3

During an assessment of a 45-year-old patient with asthma, the nurse notes wheezing and dyspnea. The nurse interprets that these symptoms are related to what pathophysiologic change? 1 Laryngospasm 2 Pulmonary edema 3 Narrowing of the airway 4 Overdistention of the alveol

Answer: 3

The nurse is assessing a patient who is having an acute asthma attack. Which nursing intervention is the priority for this patient? 1 Humidifying the room 2 Administering oral corticosteroids 3 Administering an albuterol bronchodilator 4 Placing the patient in a high-Fowler's position

Answer: 3

The nurse teaches a group of nursing students about lung volume parameter in patients with asthma. Which statement made by a student nurse indicates the need for further teaching? 1 "With asthma, there is increased residual volume." 2 "With asthma, there is increased total lung capacity." 3 "With asthma, there is increased forced expiratory volume in one second." 4 "With asthma, there is a normal forced expiratory volume to forced vital capacity ratio.

Answer: 3

Which delivery device is used for long-term oxygen therapy? 1 Nasal cannula 2 Simple face mask 3 Oxygen-conserving cannula 4 Partial and non-rebreather masks

Answer: 3

Which intervention by the nurse would be most appropriate for safe and effective care for a patient with expiratory airflow obstruction, ineffective cough, decreased airway humidity, and abnormal breath sounds? 1 Monitor for respiratory muscle fatigue. 2 Set up oxygen equipment and administer oxygen through a heated, humidified system. 3 Instruct the patient to inhale deeply, bend forward slightly, and perform three or four huffs. 4 Auscultate breath sounds to assess the areas of decreased ventilation and the presence of adventitious sounds

Answer: 3

An asthmatic patient is in acute respiratory distress. The nurse auscultates the lungs and notes cessation of inspiratory wheezing. How does the nurse interpret this finding? 1 The patient has developed a pneumothorax. 2 Airflow has now improved through the bronchioles. 3 A mucus plug has developed within a main stem bronchus. 4 There is worsening airway inflammation and bronchoconstriction

Answer: 4

The licensed practical nurse is caring for a chronic obstructive pulmonary disease (COPD) patient who has severe hypoxemia. The primary health care provider prescribed oxygen administration for this patient. Which intervention can the nurse perform for the patient? 1 Teach about home oxygen use. 2 Provide a high amount of oxygen. 3 Provide a continuous supply of oxygen. 4 Adjust the oxygen flow rate depending on the desired oxygen level

Answer: 4

The nurse is assisting a patient to learn self-administration of beclomethasone, two puffs inhaled every six hours. What should the nurse explain as the best way to prevent oral infection while taking this medication? 1 Chew a hard candy before the first puff of medication. 2 Rinse the mouth with water before each puff of medication. 3 Ask for a breath mint following the second puff of medication. 4 Rinse the mouth with water following the second puff of medication

Answer: 4

The nurse is caring for a patient who has just been diagnosed with asthma. The nurse knows what? 1 Asthma also is considered a psychosomatic disease. 2 Food allergies frequently trigger an asthma attack in adults. 3 The asthma triad refers to nasal polyps, asthma, and sensitivity to acetaminophen (Tylenol) products. 4 Gastroesophageal reflux disease (GERD) is more common in people with asthma than in the general population

Answer: 4

The nurse is teaching a patient pursed-lip breathing (PLB). What instructions by the nurse are correct? 1 "Inhale slowly and deeply through your mouth." 2 "During exhalation, relax your facial muscles and puff out your cheeks." 3 "Only use this action after any activity that causes you to be short of breath." 4 "Make breathing out (exhalation) three times longer than breathing in (inhalation)."

Answer: 4

Which measure should a patient with chronic obstructive pulmonary disorder (COPD) take when performing physical activity? 1 Walk briskly or bicycle with pursed lips. 2 Exhale while at rest and inhale while pushing. 3 Administer beta agonists immediately after exercise. 4 Remember to think about steps and breathing while walking.

Answer: 4

The nurse gathers data related to individual patients' forced expiratory volume in one-second values. The nurse suspects that which patient has intermittent asthma? 1 Patient A 2 Patient B 3 Patient C 4 Patient D

Answer: A

A 45-year-old man with asthma is brought to the emergency department by automobile. He is short of breath and appears frightened. During the initial nursing assessment, which clinical manifestation might be present as an early manifestation during an exacerbation of asthma? 1 Anxiety 2 Cyanosis 3 Bradycardia 4 Hypercapnia

Answer:1

A patient with chronic obstructive pulmonary disease (COPD) is suspected to have developed cor pulmonale. The nurse recognizes that which test result helps confirm the diagnosis? 1 Large pulmonary vessels on chest x-ray 2 Left-sided heart enlargement on echocardiogram 3 Decreased B-type natriuretic peptide (BNP) levels 4 Decreased pressure found in a right heart catheterization

Answer:1

Which complications in a patient with chronic obstructive pulmonary disorder (COPD) require acute intervention? Select all that apply. 1 Atelectasis 2 Pneumonia 3 Cor pulmonale 4 Mucoid impact 5 Exacerbations

Answer:1,2,3,5

The nurse teaches a patient regarding the administration of fluticasone through an inhaler. Which statements made by the patient indicate effective learning? Select all that apply. 1 "I have to shake the medicine bottle before use." 2 "I should keep my medicine away from humid places." 3 "I should engage the lever while loading the medicine." 4 "I should hold my breath for 20 to 30 seconds after a puff." 5 "I have to raise my head forward and breathe into my inhaler.

Answer:2,3

A patient has hyperventilation, dyspnea, nasal flaring, anxiety, and pursed-lip breathing. The nurse concludes that the patient has an ineffective breathing pattern due to alveolar hypoventilation. Which interventions should the nurse implement for this patient to promote safe and effective care? Select all that apply. 1 Avoid giving bronchodilators. 2 Monitor for respiratory muscle fatigue. 3 Encourage slow, deep breathing and coughing. 4 Elevate the head of the bed and provide an overbed table for the patient. 5 Provide factual information concerning diagnosis, treatment, and prognosis

Answer:2,3,4

A patient receives a prescription for omalizumab. What medication route of administration and schedule will the nurse include in the information provided to the patient? 1 Intravenously, every four to six hours 2 Two to four puffs, every 20 to 30 minutes 3 Subcutaneously, every two to four weeks 4 Orally, one hour before meals or two hours after meals

Answer:3

A patient who is receiving therapy for bronchial hyperresponsiveness has developed oropharyngeal candidiasis, hoarseness, and dry cough. The nurse suspects that which medication is the cause of the patient's symptoms? 1 Oral zafirlukast 2 Nebulized albuterol 3 Inhaled fluticasone 4 Subcutaneous omalizumab

Answer:3

The nurse is caring for a patient with an oxygen saturation of 45 percent of personal best who stops breathing while sleeping and has a tendency to sleep during the day. Which intervention by the nurse will help the patient most while recovering? 1 Advise the patient to walk for 30 minutes. 2 Encourage the patient to drink more fluids at mealtime. 3 Encourage the patient to use typed messages to communicate. 4 Ask the patient to speak continuously in sentences by taking deep breaths

Answer:3

A patient with a forced expiratory volume of 80 percent and a three-day history of breathlessness develops nausea, vomiting, headache, tachycardia, and dysrhythmias. The nurse suspects that which medication that the patient takes is the cause of the patient's symptoms? 1 Albuterol 2 Omalizumab 3 Mometasone 4 Theophylline

Answer:4

The nurse is teaching energy conservation techniques to a patient with severe dyspnea and oxygen saturation of 50 mm Hg. Which action of the patient indicates effective learning? 1 Standing up while performing activities 2 Using diaphragm muscles to breathe deeply 3 Inhaling a lot of air while exerting effort and exhaling at rest 4 Using a tripod posture and placing the mirror on the table while using an electric razo

Answer:4

The nurse recalls that the steps that lead to airway remodeling occur in what order? 1. Cellular infiltration 2. Neuropeptides release 3. Degranulation of mast cells 4. Inhalation of animal dander 5. Release of inflammatory mediators

Answer:4, 3, 5, 1, 2

The nurse provides education to a patient who is prescribed a metered-dose inhaler. Which actions taken by the patient indicate the need for further teaching? Select all that apply. 1 Waits between puffs 2 Activates the inhaler during inspiration 3 Holds the breath for 10 seconds after a puff 4 Inhales more than one puff with each inspiration 5 Does not shake the metered-dose inhaler before use

Answer:4, 5

Which test result identifies that a patient with asthma is responding to treatment? 1 An increase in CO2 levels 2 A decrease in exhaled nitric oxide 3 A decrease in white blood cell count 4 An increase in serum bicarbonate levels

Answer:A

A male patient with chronic obstructive pulmonary disease (COPD) becomes dyspneic at rest. His baseline blood gas results are PaO2 70 mm Hg, PaCO2 52mm Hg, and pH 7.34. What updated patient assessment requires the nurse's priority intervention?

Arterial pH 7.26

Which finding indicates to the nurse that a patient's respiratory status is improving following an acute asthma exacerbation?

Audible wheezing

Which finding indicates to the nurse that a patient's respiratory status is improving following an acute asthma exacerbation? Multiple choice question Audible wheezing Pursed lip breathing Use of intercostal muscles Oxygen saturation 89% of room air

Audible wheezing The primary problem during an exacerbation of asthma is narrowing of the airway and subsequent diminished air exchange. As the airways begin to dilate, wheezing gets louder because of better air exchange. Pursed lip breathing does not correlate with asthma improvement. The use of intercostal muscles and an oxygen saturation of 89% are evidence of continued asthma exacerbation.

A patient presents to the emergency department with acute exacerbation of asthma. What actions should the nurse perform to monitor the patient's respiratory and cardiovascular systems? Select all that apply. Multiple selection question Take a chest radiograph. Auscultate the lung sounds. Check the patient's temperature. Measure blood pressure and respiratory rate. Monitor arterial blood gases (ABGs) and pulse oximetry.

Auscultate the lung sounds. Measure blood pressure and respiratory rate. Monitor arterial blood gases (ABGs) and pulse oximetry. It is essential to monitor respiratory and cardiovascular systems in case of acute exacerbation of asthma. Auscultating lung sounds, measuring blood pressure and respiratory rate, and monitoring ABGs and pulse oximetry are required to monitor these systems. Chest radiographs are seldom useful in the management of an acute asthma attack. Checking the temperature may not contribute to monitoring respiratory and cardiovascular systems.

When teaching a patient with asthma about ways to reduce the severity of asthma and asthma attacks, which measures should be included? Select all that apply. Multiple selection question Avoid food irritants. Avoid animals with fur. Identify personal triggers. Go out in the cold air for a walk. Use nonsteroidal antiinflammatory drugs (NSAIDs).

Avoid food irritants. Avoid animals with fur. Identify personal triggers Patients with asthma should be taught to avoid food irritants and animals with fur. Identifying personal triggers can help to avoid them. Going out in the cold air and the use of NSAIDs are not recommended, because they can precipitate an asthma attack.

A patient with an acute attack of asthma is in a state of panic. Which nursing measures help to relieve the panic? Select all that apply. Multiple selection question Use sedation. Be calm, quiet, and reassuring. Encourage pursed-lip breathing. Utilize a "walking down" technique. Utilize the "talking down" technique.

Be calm, quiet, and reassuring. Encourage pursed-lip breathing. Utilize the "talking down" technique Pursed-lip breathing keeps the airways open, slows down the respiratory rate, and encourages deep breathing. "Talking down" is a technique that helps to calm the patient. A calm, quiet, and reassuring nurse helps to pacify the patient. Use of sedatives should not be encouraged, because they may cause respiratory depression. There is no technique called "walking down."

The nurse observes that a patient with cystic fibrosis (CF) has a body temperature of 102° F, shortness of breath, sore throat, and purulent mucous. The nurse expects what other assessment finding?

Because of the CF, the presence of bullae and blebs in the lungs can also be observed. The symptoms the patient is experiencing, specifically a body temperature of 102° F, shortness of breath, sore throat, and purulent mucus, are indicative of a chronic lung infection. The patient with advanced lung disease (versus a lung infection) has digital clubbing. A patient with bloody sputum and dyspnea, fever, chills, and chest pain is experiencing bronchiectasis. A patient with an accumulation of mucus in the liver experiences chronic elevation of liver enzymes, which leads to cirrhosis.

The nurse recognizes that which treatment may provide relief to a patient with asthma that has a PaO2 of 60 mm Hg and dyspnea?

Bronchial thermoplasty

The nurse expects that what diagnosis will be made for a patient that has permanent, abnormal dilation of medium-sized bronchi, inflammation, and elastic and muscular structural destruction of the bronchial wall?

Bronchiectasis

During an assessment of a 45-year-old patient with asthma, the nurse notes wheezing and dyspnea. The nurse determines that these symptoms are related to which pathophysiologic feature of the disease?

Bronchoconstriction

The nurse identifies the nursing diagnosis of "ineffective airway clearance" for a 58-year-old patient with asthma. The nurse should assess for which common etiologic factor for this nursing diagnosis?

Bronchospasm

The nurse is preparing a patient for a procedure with the thoracoscope to reduce the exacerbations of chronic obstructive pulmonary disease (COPD). What procedure should the nurse be sure the patient understands?

Bullectomy

A patient with chronic obstructive pulmonary disease (COPD) is experiencing anxiety. What medication as ordered should the nurse administer to this patient?

Buspirone

The nurse is collecting data on four patients with a history of mild asthma. Which patient is most likely to experience wheezing, congestion and angioedema? A - Hypertension; lisinopril B - Glaucoma; takes timolol C - Dysmenorrhea, takes iburprofen D - GERD, epinephrine

C - Dysmenorrhea, takes iburprofen Patient C, who has dysmenorrhea and takes ibuprofen, is more likely to have wheezing within two hours of drug administration. In addition, the patient usually presents with profound rhinorrhea, congestion, tearing, and angioedema. Patient A, who has hypertension and takes lisinopril, may have a cough, which exacerbates asthma. Patient B who has glaucoma and who is taking timolol, may experience bronchospasm. Patient D, who has gastroesophageal reflux disease and is taking epinephrine, has a low incidence of asthma.

What is an autosomal recessive, multisystem disease characterized by altered function of the exocrine glands?

CF is an autosomal recessive, multisystem disease characterized by altered function of the exocrine glands. This defect primarily affects the lungs, pancreas and biliary tract, and sweat glands. Sweat glands excrete increased amounts of sodium and chloride. While sickle-cell disease, Tay-Sachs disease, and spinal muscular atrophy are all autosomal recessive, multisystem diseases; they are not characterized by altered function of the exocrine glands.

The nurse finds that a patient is experiencing breathlessness, chest tightness, and a cough after climbing stairs. What is the most likely reason for these symptoms?

Capillary leakage in the airway

The nurse finds that a patient is experiencing breathlessness, chest tightness, and a cough after climbing stairs. What is the most likely reason for these symptoms? Multiple choice question Edema of airway walls Capillary leakage in the airway Change in the responsiveness of airways Immunoglobulin E-mediated allergic response

Capillary leakage in the airway Asthma that is induced or exacerbated during physical exertion is called exercise-induced asthma (EIA). Typically, EIA occurs after vigorous exercise, not during it. Airway obstruction may occur due to changes in the airway mucosa caused by hyperventilation during exercise, with either cooling or rewarming of air and capillary leakage in the airway wall. Edema of airway walls occurs during respiratory tract infections. A change in responsiveness of the airway occurs on exposure to irritants, as seen with occupational asthma. Patients who are genetically predisposed to develop an allergic (immunoglobulin E-mediated) response have an increased risk of allergic asthma, but it would not cause an exacerbation after climbing stairs.

A patient experiences chronic sinusitis. The nurse recognizes that which treatment strategy may help relieve the patient's symptoms caused by impaired ciliary movement in the airway?

Chest wall oscillation

A patient with chronic obstructive pulmonary disease (COPD) is receiving oxygen therapy through a mask. Which nursing actions should the nurse perform to ensure proper care of the patient? Select all that apply.

Choosing the optimal oxygen device, assessing the need for adjustment in oxygen flow rate, and monitoring signs of adverse effects of oxygen therapy are all duties that the nurse should perform when evaluating the response of the patient to oxygen therapy. Taking a chest radiograph and assessing the need to change intravenous fluids are not relevant to this situation.

A patient in the outpatient clinic has symptoms including chronic cough, sputum production, and dyspnea. On taking a detailed history of the patient, the nurse finds that this patient has a prolonged exposure to smoke. Which condition would the nurse most likely suspect the patient to have?

Chronic obstructive pulmonary disease (COPD)

Which corticosteroid can be administered directly through inhalation? Multiple choice question Fluticasone Ciclesonide Budesonide Mometasone

Ciclesonide Ciclesonide has reduced local side effects like oropharyngeal candidiasis, hoarseness, and dry cough because it activates the lungs and is administered directly through inhalation. Drugs such as fluticasone, budesonide, and mometasone cause local irritation as they are activated in the pharynx. Therefore these medications require a spacer for delivery into the lungs.

The nurse provides education to a patient with asthma about how to take medication through a metered dose inhaler. Which action performed by the patient indicates effective learning?

Cleans the device with water

The nurse provides education to a patient with asthma about how to take medication through a metered dose inhaler. Which action performed by the patient indicates effective learning? Multiple choice question Cleans the device with water Inhales two puffs, two seconds apart Does not shake the medication before taking it Breathes in rapidly while taking the medication

Cleans the device with water Most of the medications used in the treatment of asthma are delivered through inhalers like metered dose inhalers. This helps to prevent systemic side effects and promote the onset of action. The patient should clean the plastic case of the metered dose inhaler with water. The patient should inhale two puffs per dose and shake the device before use. The patient should breathe slowly for at least 10 seconds after administration of a puff.

The nurse determines that the patient has experienced the full benefits of medication therapy with ipratropium when which assessment finding is noted?

Clear lung sounds

The nurse determines that the patient has experienced the full benefits of medication therapy with ipratropium when which assessment finding is noted? Multiple choice question Clear lung sounds Heart rate 80 beats/minute Capillary refill less than three seconds Positive bowel sounds in all quadrants

Clear lung sounds Ipratropium is an inhaled anticholinergic used for asthma management. Clear lung sounds would indicate full passage of air and well-controlled symptom management. Heart rate, capillary refill, and bowel sounds are not associated with the benefits of ipratropium administration for bronchoconstriction and inflammation.

The nurse concludes that interventions carried out to promote airway clearance in a patient admitted with asthma are successful on the basis of which finding?

Clearance of mucous from the bronchi

The nurse is assessing a patient who may have manifestations of chronic obstructive pulmonary disease (COPD). Which of these is a clinical manifestation of early COPD?

Clinical manifestations of COPD typically develop slowly. A chronic intermittent cough, which is often the first symptom to develop, later may be present every day as the disease progresses. Typically, dyspnea is progressive, usually occurs with exertion, and is present every day. Dyspnea at rest and chest breathing are manifestations of late COPD. The cough may be unproductive of mucus.

On examining a patient with asthma the nurse finds that the patient experiences asthmatic symptoms throughout the day, besides experiencing night-time awakenings more than four times a week. The patient's forced expiratory volume in the first second of expiration (FEV1) is less than 60%, and normal activity is very limited. Which treatment option should the nurse consider appropriate?

Consider oral corticosteroids

On examining a patient with asthma the nurse finds that the patient experiences asthmatic symptoms throughout the day, besides experiencing night-time awakenings more than four times a week. The patient's forced expiratory volume in the first second of expiration (FEV 1) is less than 60%, and normal activity is very limited. Which treatment option should the nurse consider appropriate? Multiple choice question Follow up after a month. Consider oral corticosteroids. Reevaluate in two to six weeks. Advise maintaining control of asthma symptoms.

Consider oral corticosteroids. Experiencing symptoms of asthma throughout the day and also experiencing nighttime awakenings more than four times a week is suggestive of poorly controlled asthma. This patient also has an FEV 1 less than 60%; normal activity being very limited correlates with that. Treatment with oral corticosteroids should be considered. Advice for maintaining control of asthma symptoms and following up after a month can be given in case of well controlled asthma. Reevaluation in two to six weeks is suggested if the asthma is not well controlled or if the patient experiences symptoms more than two times a month.

A 61-year-old patient with asthma is admitted to the hospital. The nurse understands that symptoms of asthma include what? Select all that apply. Multiple selection question Cough Crackles Wheezing Chest tightness Pink frothy sputum

Cough Wheezing Chest tightness Symptoms of asthma include cough, chest tightness, and wheezing. Crackles are heard when fluid has accumulated in the lungs, which is not consistent with asthma. Pink frothy sputum is seen with pulmonary edema.

Which change occurs during cystic fibrosis?

Cystic fibrosis (CF) affects both the upper and lower respiratory tracts. It affects the small airway first and progresses to the larger airways, finally passing into the lungs, causing hyperinflation of the lungs due to obstruction of the bronchioles by thick mucus that traps the air. The mucous dehydrates and becomes thick due to decreased ciliary movement. One of the first signs is a strong salty taste to the skin. This is the result of higher levels of salt present in the sweat. Parents of children with cystic fibrosis have mentioned tasting this saltiness when kissing their children. A hallmark of CF in children is poor weight gain and growth. These children are unable to get enough nutrients from their food because of the lack of enzymes to help absorb fats and proteins

The nurse, who has administered a first dose of oral prednisone to a patient with asthma, writes on the care plan to begin monitoring for which patient parameters? Multiple choice question Apical pulse Daily weight Bowel sounds Deep tendon reflexes

Daily weight Corticosteroids such as prednisone can lead to weight gain. For this reason, it is important to monitor the patient's daily weight. The drug should not affect the apical pulse, bowel sounds, or deep tendon reflexes.

When teaching the patient with chronic obstructive pulmonary disease (COPD) about smoking cessation, what information should be included related to the effects of smoking on the lungs and the increased incidence of pulmonary infections?

Decreased alveolar macrophage function

A patient who has a family history of α1-protease inhibitor deficiency reports fever of unknown cause, malaise, and cough associated with purulent sputum. Which test should the patient undergo regularly to assess the severity of the condition?

Deficiency of α1-antitrypsin (AAT) is an autosomal recessive disorder that affects the lungs or liver. AAT deficiency is a genetic risk factor for chronic obstructive pulmonary disease (COPD). The patient with a family history of α1-protease inhibitor deficiency should consult a pulmonologist about regular spirometry screening. This helps the patient get appropriate genetic counseling. Liver enzyme tests, renal hormone tests, and computer tomography may not provide complete relevant information to counsel the patient.

The nurse would monitor which comorbidity in the patient treated for an asthma exacerbation with methylprednisolone? Multiple choice question Hyperlipidemia Hypothyroidism Diabetes mellitus Raynaud's phenomenon

Diabetes mellitus Hyperglycemia or increased blood glucose level is an adverse effect of methylprednisolone, so the patient with diabetes mellitus should be monitored for elevations in blood sugar. Methylprednisolone will not affect elevated cholesterol, hypothyroidism, or Raynaud's phenomenon.

The nurse is educating a patient regarding breathing techniques. What technique should the nurse avoid in a patient with chronic obstructive pulmonary disease (COPD) who has marked hyperinflation?

Diaphragmatic breathing

A patient is hospitalized with a cough, weight loss, increased sputum, and decreased pulmonary function. The patient reports pain in the lower right quadrant, nausea, and emesis. The nurse suspects that the patient is experiencing what condition?

Distal intestinal obstructive syndrome

The nurse is providing postural drainage therapy on a patient who has cystic fibrosis. Postural drainage is used to do what?

Drain secretions from specific segments of the lungs and bronchi into the trachea

A patient with asthma is prescribed ipratropium bromide. The nurse recognizes that the patient may develop what side effect of the medication? Multiple choice question Anxiety Insomnia Dry cough Dry mouth

Dry mouth A patient who is taking ipratropium bromide may develop dry mouth due to inhibition of parasympathetic nervous system. Anxiety is a side effect in the patient who is taking an oral medication of a beta-adrenergic agonist. Insomnia is the common side effect of methylxanthine. Corticosteroids upon inhalation may produce local irritation, such as dry cough.

A male patient experiences a cough, perfuse sweating, local hypoxia, and arteriolar vasoconstriction. The patient states, "I hope that this condition allows me to have biologic children." What is the best nursing response?

Educate the patient about assisted reproductive technology

The nurse concludes that nursing interventions to promote airway clearance in a patient admitted with chronic obstructive pulmonary disease (COPD) are successful based on which finding?

Effective and productive coughing

A patient with wheezing, coughing, and frequent pneumonia experiences large, oily, and frequent bowel movements. The patient's medical history reveals poor absorption of fats and proteins. The nurse anticipates what assessment finding?

Emaciated appearance of the extremities

A patient with asthma experiences anaphylaxis. Which medication should the nurse prepare to administer?

Epinephrine

The nurse is assigned to care for a patient who has anxiety and an exacerbation of asthma. What is the primary reason for the nurse to carefully inspect the chest wall of this patient?

Evaluate the use of intercostal muscles

When admitting a patient with a diagnosis of asthma exacerbation, the nurse will assess for what potential triggers? Select all that apply. Multiple selection question Exercise Allergies Emotional stress Decreased humidity Upper respiratory infections

Exercise Allergies Emotional stress Upper respiratory infections Although the exact mechanism of asthma is unknown, there are several triggers that may precipitate an attack. These include allergens, exercise, air pollutants, upper respiratory infections, drug and food additives, psychologic factors, and gastroesophageal reflux disease (GERD). Decreased humidity is not a trigger.

When teaching a patient about chronic obstructive pulmonary disease (COPD) rehabilitation, what strategy should the nurse teach the patient as essential to perform for energy conservation?

Exercise training

A patient with allergic asthma has been prescribed omalizumab. The patient requests that the medication be administered at home for convenience. Which nursing action is appropriate in this case? Multiple choice question Administer the injection to the patient on a home visit. Ask a family member to administer the medication at home. Load the injection and provide it to the patient for self-administration. Explain to the patient that the medication should be administered strictly at the clinic.

Explain to the patient that the medication should be administered strictly at the clinic. Omalizumab is a monoclonal antibody to IgE that decreases circulating free IgE levels. The drug prevents IgE from attaching to mast cells, preventing the release of chemical mediators that may exacerbate asthma. The medication can cause anaphylactic reaction and should be administered at a clinic that is well-equipped to handle emergencies. The nurse should not encourage self-administration of the medication at home, because the anaphylactic reaction can be life-threatening. The nurse may not be well equipped to handle emergencies at home, so the medication should not be administered during a home visit. A family member should not be asked to administer the medication for the same reason.

The nurse is reviewing the diagnostic reports of a patient admitted for assessment of respiratory issues. Which diagnostic finding would confirm a diagnosis of chronic obstructive pulmonary disease (COPD)?

FEV1/FVC ratio of 65%

Nursing assessment findings of jugular venous distention and pedal edema would be indicative of what complication of chronic obstructive pulmonary disease (COPD)?

Fluid volume excess resulting from cor pulmonale

The nurse observes that a patient with cystic fibrosis (CF) has a body temperature of 102° F, shortness of breath, sore throat, and purulent mucous. The nurse expects what other assessment finding?

Formation of bullae and blebs in lungs

A patient with intermittent obstruction of the terminal ileum experiences thickened, dehydrated stools and increased mucus production. Subsequently, the patient develops chronic malabsorption due to partial bowel obstruction. The nurse anticipates that which intervention will be included on the patient's plan of care?

Gastric decompression and ingesting polyethylene glycol

The nurse is caring for a patient who has just been diagnosed with asthma. The nurse knows what?

Gastroesophageal reflux disease (GERD) is more common in people with asthma than in the general population.

The nurse is caring for a patient who has just been diagnosed with asthma. The nurse knows what? Multiple choice question Asthma also is considered a psychosomatic disease. Food allergies frequently trigger an asthma attack in adults. The asthma triad refers to nasal polyps, asthma, and sensitivity to acetaminophen (Tylenol) products. Gastroesophageal reflux disease (GERD) is more common in people with asthma than in the general population.

Gastroesophageal reflux disease (GERD) is more common in people with asthma than in the general population. GERD occurs more often in people with asthma than in the general population. Asthma is not a psychosomatic disease, although symptoms can worsen with stress. A food allergy triggering an asthma attack in adults is rare. The asthma triad refers to nasal polyps, asthma, and sensitivity to aspirin and nonsteroidal antiinflammatory drugs (NSAIDs).

A patient with emphysema is receiving oxygen at 1 L/min by way of nasal cannula. Why does the nurse understand that this prescription is appropriate?

High concentrations of oxygen eliminate the respiratory drive

Which laboratory abnormality observed by the nurse confirms the diagnosis of a patient with low body mass index, wheezing, coughing, frequent pneumonia, bleeding, and sweating?

High sweat chloride level

A 45-year-old patient is experiencing an asthma exacerbation. To facilitate airflow, the nurse should place the patient in which position?

High-Fowler's

In which position is it most appropriate for the nurse to place a patient experiencing an asthma exacerbation?

High-Fowler's

In which position is it most appropriate for the nurse to place a patient experiencing an asthma exacerbation? Multiple choice question Supine Lithotomy High-Fowler's Reverse Trendelenburg

High-Fowler's The patient experiencing an asthma attack should be placed in high-Fowler's position and may need to lean forward to allow for optimal chest expansion and enlist the aid of gravity during inspiration. The supine, lithotomy , and reverse Trendelenburg positions will not facilitate ventilation.

A patient experiences pleuritic chest pain, dyspnea, wheezing, clubbing of digits, weight loss, and blood streaked sputum. The nurse anticipates that which test will be prescribed?

High-resolution computer tomography

To teach effective huff coughing to a patient with chronic obstructive pulmonary disease (COPD), the nurse should instruct the patient to perform which action?

Huff coughing is completed through forceful exhalations to rid the lungs of mucus once it is felt in the bronchi. The patient using upright, abdominal splinting is not indicated, and the patient should not hold the breath before coughing.

When planning teaching for the patient with chronic obstructive pulmonary disease (COPD), the nurse understands that what causes the manifestations of the disease?

Hyperinflation of alveoli and destruction of alveolar walls

What are the structural changes that occur during chronic inflammation? Select all that apply. Multiple selection question Fibrosis of epithelium Smooth muscle atrophy Hypersecretion of mucus Progressive loss of lung function Proliferation of new blood vessels

Hypersecretion of mucus Progressive loss of lung function Proliferation of new blood vessels Chronic inflammation of the airway results in remodeling of the bronchial wall. This includes hypersecretion of mucus, progressive lung function loss, and proliferation of new blood vessels. The changes may also include fibrosis of the subepithelium and smooth muscle hypertrophy of the airways.

The nurse is providing education about the risk factors of asthma. Which factors does the nurse explain are associated with asthma? Select all that apply. Multiple selection question Immune response Sedentary lifestyle History of pancreatitis Genetic predisposition Exposure to air pollutants

Immune response Genetic predisposition Exposure to air pollutant Risk factors for asthma include immune response, genetic predisposition, and exposure to air pollutants. Exercise, not a sedentary lifestyle, is also a risk factor. A history of gastroesophageal reflux disease is a risk factor, but a history of pancreatitis is not.

When teaching a patient about using a dry powder inhaler, what instructions should be included in this teaching? Select all that apply. Multiple selection question Increase the inspiration. Slow down the inspiration. Keep the device moisture-free. Shake the inhaler well before use. Avoid shaking the inhaler before use.

Increase the inspiration. Keep the device moisture-free. Avoid shaking the inhaler before use With dry powder inhalers, there is no need to shake before use. Inspiration should be rapid, and the device should be kept moisture-free to protect the dry powder. Shaking the inhaler well before use and slow inspiration are actions performed when using metered-dose inhalers

The nurse is assessing a patient with asthma who has been diagnosed with a severe and life-threatening exacerbation. What findings would the nurse find? Multiple choice question Increased CO 2 level Speaking in short sentences Increased pH level on an arterial blood gas (ABG) Peak expiratory flow rate (PEFR) is 70% of the personal best

Increased CO 2 level Early in exacerbation the CO 2 level is decreased, but increases if exacerbation is prolonged or severe. With an asthma exacerbation, patients are unable to speak in sentences and only speak a few words at a time before taking a breath. The pH level on an ABG decreases as the episode is prolonged, but is increased early in the exacerbation. With a life-threatening asthma attack, PEFR is 40% of the patient's personal best.

A patient who is receiving therapy for bronchial hyperresponsiveness has developed oropharyngeal candidiasis, hoarseness, and dry cough. The nurse suspects that which medication is the cause of the patient's symptoms?

Inhaled fluticasone

The nurse provides education to a patient who is prescribed a metered-dose inhaler. Which actions taken by the patient indicate the need for further teaching? Select all that apply. Multiple selection question Waits between puffs Activates the inhaler during inspiration Holds the breath for 10 seconds after a puff Inhales more than one puff with each inspiration Does not shake the metered-dose inhaler before use

Inhales more than one puff with each inspiration Does not shake the metered-dose inhaler before use The metered-dose inhaler (MDI) has to be shaken before use, and the patient should only inhale one puff per inspiration. The patient using an MDI should wait between each puff. The MDI should be activated during inspiration. The patient should to hold the breath for 10 seconds after each puff.

What is a priority nursing assessment for a 38-year-old patient experiencing an acute asthma exacerbation? Multiple choice question Pupillary response to light Inspection of the chest wall Measurement of pedal pulses Percussion for costovertebral angle (CVA) tenderness

Inspection of the chest wall The nurse physically inspects the chest wall to evaluate the use of intercostal muscles, which gives an indication of the degree of respiratory distress experienced by the patient. Pupillary response is a neurologic, not respiratory, assessment. Pedal pulses are measured to assess circulatory function. CVA tenderness is indicative of kidney inflammation, not asthma

A child with a nonproductive cough states, "I am having trouble breathing." What action should the nurse take to reduce the severity of breathlessness in the child?

Instruct the child to bend forward slightly

Which intervention by the nurse would be most appropriate for safe and effective care for a patient with expiratory airflow obstruction, ineffective cough, decreased airway humidity, and abnormal breath sounds?

Instruct the patient to inhale deeply, bend forward slightly, and perform three or four huffs.

After the inhalation of puffs of mometasone, a patient develops oropharyngeal candidiasis, hoarseness and dry cough. What action should the nurse take to reduce the symptoms?

Instruct the patient to rinse the mouth with water after inhalation

After the inhalation of puffs of mometasone, a patient develops oropharyngeal candidiasis, hoarseness and dry cough. What action should the nurse take to reduce the symptoms? Multiple choice question Recommend that the patient pauses between the puffs Instruct the patient to rinse the mouth with water after inhalation Assist the patient in obtaining a spacer or holding device for inhalation Wait until the cough subsides before administering the patient's next dose

Instruct the patient to rinse the mouth with water after inhalation Upon inhalation into the pharynx, mometasone may cause local irritation such as oropharyngeal candidiasis, hoarseness, and dry cough. Hence the patient should rinse the mouth either with water or with mouthwash after inhalation. The patient may not be benefit by pausing between the puffs. Asking the patient to use a spacer or holding device for inhalation of corticosteroids can be helpful in getting more medication into the lungs. However, it does not reduce the symptoms of candidiasis. The next dose is given to the patient only upon further advice from the practitioner.

The nurse recognizes that which patient condition is associated with a sweat chloride level of 75 mmol/L and chromosome 7 mutation's effect on the development of bone?

Insufficient testosterone levels

A patient with cystic fibrosis (CF) is hospitalized with exacerbation of symptoms, wheezing, purulent sputum, and a fasting blood glucose level of 194 mmol/L. The nurse anticipates that what medication will be prescribed?

Insulin

A patient experiencing an acute asthma exacerbation has received a bronchodilator and supplemental oxygen. Which treatment should the nurse anticipate if the patient's condition remains unchanged?

Intravenous corticosteroids

The nurse recalls that which type of asthma involves an acute asthma attack triggered by an upper respiratory infection? Multiple choice question Intrinsic Allergic Extrinsic Emotional

Intrinsic Intrinsic asthma does not have an easily identifiable allergen and may be related to internal factors such as an upper respiratory infection. Extrinsic asthma includes allergic triggers such as dust, molds, and pet dander. Asthma related to emotional causes is also considered extrinsic.

Which measure should a patient with chronic obstructive pulmonary disorder (COPD) take when performing physical activity?

It is essential to involve the patient with chronic obstructive pulmonary disease (COPD) in physical activity for 20 minutes three times a week. The patient should remember to think about steps and breathing, which will help decrease anxiety and also slow the pace. The patient must walk slowly with pursed lips to breathe. The patient should exhale while pushing and inhale while at rest. The patient should wait for 5 minutes to relax and regain a normal breathing rate before administration of a beta agonist after exercise.

What are the advantages of a diskus over a metered-dose inhaler? Select all that apply. Multiple selection question It requires less manual dexterity. It includes coordination of device puffs with inhalation. The medication does not clump upon exposure to the humidity. It contains an easily visible color that indicates the number of doses left. A patient with low forced expiratory volume in one second can easily inhale the medication using the diskus.

It requires less manual dexterity It contains an easily visible color that indicates the number of doses left. A diskus has several advantages over metered-dose inhaler. There is less need for manual dexterity through a diskus. The visible color indicator on a diskus helps to estimate the number of doses left. There is no need to coordinate device puffs with inhalation. The medication in the diskus may clump upon exposure to the humid environment. The patient has to apply inspiratory effort to administer the drug. Therefore the patient who has low forced expiratory volume in one second cannot inhale the medication.

While teaching a patient with asthma about the appropriate use of a peak flow meter, what should the nurse instruct the patient to do? Multiple choice question Keep a record of the peak flow meter numbers if symptoms of asthma are getting worse. Use the flow meter each morning after taking medications to evaluate their effectiveness. Increase the doses of the long-term control medication if the peak flow numbers decrease. Empty the lungs and then inhale quickly through the mouthpiece to measure how fast air can be inhaled.

Keep a record of the peak flow meter numbers if symptoms of asthma are getting worse. It is important to keep track of peak flow readings daily, especially when the patient's symptoms are getting worse. The patient should have specific directions as to when to call the health care provider based on personal peak flow numbers. Peak flow is measured by exhaling into the flow meter, and should be assessed before and after medications to evaluate their effectiveness.

A patient with chronic obstructive pulmonary disease (COPD) is suspected to have developed cor pulmonale. The nurse recognizes that which test result helps confirm the diagnosis?

Large pulmonary vessels on chest x-ray

A patient with chronic obstructive pulmonary disease (COPD) is suspected to have developed cor pulmonale. The nurse recognizes that which test result helps confirm the diagnosis? Multiple choice question Large pulmonary vessels on chest x-ray Left-sided heart enlargement on echocardiogram Decreased B-type natriuretic peptide (BNP) levels Decreased pressure found in a right heart catheterization

Large pulmonary vessels on chest x-ray Cor pulmonale is a cardiac complication of COPD resulting from pulmonary hypertension. Due to pulmonary hypertension, the pulmonary vessel may appear enlarged in a chest x-ray. There may be increased pressure found in a right heart catheterization due to pulmonary hypertension. Cor pulmonale is usually associated with right-sided heart enlargement, because there is increased pressure in the blood vessels of lungs. The BNP levels are increased due to the stretching of the right ventricle.

A patient with dyspnea and hypoxemia has received an initial nebulized short-acting β 2-adrenergic agonist (SABA) with ipratropium treatment. The patient's forced expiratory volume in one second is 60 percent, and the peak flow is less than 25 percent of personal best. The nurse anticipates that which medication will be administered? Multiple choice question Epinephrine Theophylline Magnesium sulfate Sodium bicarbonate

Magnesium sulfate A patient with dyspnea and hypoxemia and who is on initial treatment with ipratropium has severe asthma. Such a patient has low forced expiratory volume in one second of 60 percent and peak flow less than 25 percent of personal best. Intravenous administration of magnesium sulfate helps to resolve the patient's condition faster. Administration of epinephrine is not indicated for the treatment of asthma exacerbations. However, a subcutaneous or intramuscular injection may be used for acute treatment of anaphylaxis. Administration of theophylline is no longer recommended for asthma exacerbations. Sodium bicarbonate administration is limited to the treatment of severe metabolic or respiratory acidosis.

A patient is concerned that he or she may have asthma. The nurse assesses the severity of the symptoms. Based on this information, how would the nurse classify the patient's asthma? Refer to chart.

Mild persistent

An asthmatic patient was prescribed theophylline. A nurse understands that the patient is at risk for tachycardia and seizures. In regard to safety, the nurse expects that what will be included on the patient's treatment plan? Multiple choice question Encourage the use of caffeine. Use diazepam to prevent seizures. Monitor serum blood levels of adrenaline. Monitor serum blood levels of theophylline.

Monitor serum blood levels of theophylline. Tachycardia and seizures are known toxic effects of theophylline at higher blood levels. In addition, the drug has a narrow margin of safety. Therefore monitoring blood levels of theophylline helps to reduce such toxic effects. Caffeine increases the toxic effects of theophylline. Monitoring plasma levels of adrenaline may not help, because tachycardia is not associated with adrenaline in this case. Prophylactic use of diazepam may not prevent seizures that are caused by the use of theophylline.

During an assessment of a 45-year-old patient with asthma, the nurse notes wheezing and dyspnea. The nurse interprets that these symptoms are related to what pathophysiologic change?

Narrowing of the airway

During an assessment of a 45-year-old patient with asthma, the nurse notes wheezing and dyspnea. The nurse interprets that these symptoms are related to what pathophysiologic change?

Narrowing of the airway by persistent but variable inflammation leads to reduced airflow, making it difficult for the patient to breathe and producing the characteristic wheezing. Laryngospasm, pulmonary edema, and overdistention of the alveoli do not produce wheezing.

Which complication does the nurse monitor in the patient who is using an oxygen-conserving cannula?

Necrosis over tops of ears

The nurse is caring for a patient with cystic fibrosis (CF). What is the most important factor for the nurse to consider when using clearance techniques and devices?

No single technique has shown superiority over the others

The nurse is caring for the patient with cystic fibrosis (CF). Which is the most important factor for the nurse to consider?

Of the CF patients in the Cystic Fibrosis Patient Registry, more than 47% are 18 years of age or older. The median predicted survival was 16 years in 1970, but has increased to more than 38 years. With early diagnosis and improvements in therapy, the prognosis of patients with CF has improved significantly. Nurses who work in adult care settings increasingly manage patients with CF. The severity and progression of the disease varies and the first signs and symptoms typically occur in children. However, some patients are not diagnosed until they are adults; some live to over 80 years old.

The nurse determines that a patient is experiencing common adverse effects from the inhaled corticosteroid beclomethasone after what occurs?

Oropharyngeal candidiasis and hoarseness

The nurse determines that a patient is experiencing common adverse effects from the inhaled corticosteroid beclomethasone after what occurs? Multiple choice question Hypertension and pulmonary edema Oropharyngeal candidiasis and hoarseness Elevation of blood glucose and calcium levels Adrenocortical dysfunction and hyperglycemia

Oropharyngeal candidiasis and hoarseness Oropharyngeal candidiasis and hoarseness are common adverse effects from the use of inhaled corticosteroids because the medication can lead to overgrowth of organisms and local irritation if the patient does not rinse the mouth following each dose. Beclomethasone does not cause hypertension, pulmonary edema, elevated calcium levels or blood glucose levels, or adrenocortical dysfunction.

The nurse is caring for a five-year-old child with low body weight and poor growth. The parent reports that the child is unable to digest fat- and protein-rich foods. The nurse suspects that the child has what condition?

Pancreatic insufficiency

A patient with cystic fibrosis (CF) has a body mass index (BMI) of 15.6 kg/m 2 and experiences frequent, bulky, and foul-smelling stools? What does the nurse expect to find on the patient's plan of care?

Pancrelipase before each meal

Which laboratory finding helped the nurse reach the conclusion that a patient with a chronic cough and dyspnea has hypercapnia?

Partial pressure of carbon dioxide (PaCO2) is 55 mm Hg

Which finding helped the nurse reach the conclusion that a patient with chronic obstructive pulmonary disease (COPD) requires oxygen therapy?

Partial pressure of oxygen (PaO2) 52 mm Hg

The nurse gathers data related to individual patients' forced expiratory volume in one-second values. The nurse suspects that which patient has intermittent asthma?

Patient A

The nurse assesses four patients and recognizes that which patient is likely to experience small bowel obstruction?

Patient B

The registered nurse (RN) working on a pulmonary floor is delegating tasks to a licensed practical nurse (LPN). Which patient intervention can appropriately be delegated to the LPN?

Patient B

Which intervention is beneficial to a patient with chronic obstructive pulmonary disease (COPD)?

Patients with chronic obstructive pulmonary disease (COPD) should limit fluid intake during mealtimes because too much liquid might make the patient feel too full to eat. COPD patients should eat cold foods rather than hot foots in order to feel less full. COPD patients should eat frequent meals and snacks because it helps the diaphragm move freely and makes gas exchange in the lungs easier. Performing physical activity before meals may increase breathlessness and may affect food intake.

Which strategy should the nurse teach a patient with emphysema to manage anxiety related to impaired gas exchange?

Perform pursed-lip breathing

A patient with cystic fibrosis (CF) experiences wheezing, coughing, and frequent pneumonia. The patient's computer tomography shows bullae and blebs of 1 to 2 cm in the lungs. The nurse anticipates that what will be included on the patient's plan of care?

Pleural abrasion

Which condition may develop if a patient with localized hypoxia and arteriolar vasoconstriction who has blebs and large cysts in the lungs is left untreated?

Pneumothorax

A patient reports severe abdominal pain, and the laboratory report indicates accumulation of mucus and high levels of liver enzymes. Which condition may develop if the patient is left untreated?

Portal hypertension

A nurse is caring for a patient who is having an acute asthma attack. Which interventions should the nurse question? Select all that apply. Multiple selection question Peak flow reading Arterial blood gases Prednisone 10 mg, PO X1 stat Continuous oxygen to keep oxygen saturation above 90% Administration of blood products to increase oxygen saturation

Prednisone 10 mg, PO X1 stat Administration of blood products to increase oxygen saturation The nurse should question prednisone 10 mg, PO because this is an urgent situation that requires intravenous, intramuscular, and inhaled medications, rather than oral ones. There is not enough documentation to indicate that this patient requires administration of blood products, and blood products are not used to treat oxygen saturation. Peak flow readings, arterial blood gases, and continuous oxygen would be considered the standard of care for this patient.

The nurse teaches pursed-lip breathing to a patient with emphysema. The nurse explains that the primary reason for this technique is what?

Promoting carbon dioxide elimination

A nurse has taught the technique of pursed-lip breathing to a patient. During a return demonstration, what patient action requires correction? Multiple choice question Puffing of cheeks while exhaling air Slow and deep inhalation through the nose Slow exhalation through pursed lips, as if whistling Exhalation time three times as long as inhalation time

Puffing of cheeks while exhaling air In pursed-lip breathing, the patient should avoid puffing of the cheeks while exhaling the air. Puffing of the cheeks makes the technique less effective. Slow and deep inhalation, slow exhalation through pursed lips as if whistling, and exhalation time thrice as long as inhalation time are correct techniques of pursed-lip breathing.

The nurse determines that the patient with chronic obstructive pulmonary disease (COPD) and diabetes mellitus is experiencing adverse effects of albuterol after noting which finding? Multiple choice question Blood sugar 139 mg/dL Temperature of 99.1° F Respiratory rate of 21 breaths/minute Pulse rate of 102 beats/minutes

Pulse rate of 102 beats/minutes Albuterol is a β2-agonist that sometimes can cause adverse cardiovascular effects. These would include tachycardia and angina. A pulse rate of 102 indicates that the patient is experiencing tachycardia as an adverse effect. This medication will not affect the blood sugar or the temperature. The respiratory rate is normal and does not indicate any adverse reaction is occurring.

The nurse determines that the patient is not experiencing adverse effects of albuterol after noting which patient vital sign?

Pulse rate of 72 beats/minute

The purpose of this exercise is to prolong exhalation and thereby prevent bronchiolar collapse and air trapping.

Pursed-lip breathing (PLB)

A patient experiences exacerbations of wheezing and breathlessness when exercising in the early morning. The nurse learns that patient has a three-year history of sinusitis. What is appropriate to be included on the patient's plan of care?

Referring the patient to a surgeon for the removal of nasal polyps

A patient experiences exacerbations of wheezing and breathlessness when exercising in the early morning. The nurse learns that patient has a three-year history of sinusitis. What is appropriate to be included on the patient's plan of care? Multiple choice question Prescribing a beta-antagonist for the patient Administering intravenous corticosteroids to the patient Advising the patient to exercise when the air is cool and dry Referring the patient to a surgeon for the removal of nasal polyps

Referring the patient to a surgeon for the removal of nasal polyps Both acute and chronic sinusitis make asthma worse, causing severe inflammation of the mucous membranes. Such patients should undergo removal of large nasal polyps to help control exacerbations of asthma. Beta-antagonists promote bronchoconstriction, leading to further airway obstruction and breathlessness in the patient. The nurse should advise the patient to avoid exercise when the air is cool and dry to prevent exacerbations. Intravenous corticosteroids are as effective as inhaled corticosteroids. However, the patient may experience exacerbations upon administration of corticosteroids intravenously. This intervention may increase fluid accumulation in the sinuses, further leading to pain.

Which measure should a patient with chronic obstructive pulmonary disorder (COPD) take when performing physical activity?

Remember to think about steps and breathing while walking

Before discharge, the nurse discusses nutrition with the patient with emphysema and pneumonia. The nurse instructs the patient to do what?

Rest for 30 minutes before eating

The patient has been receiving oxygen by nasal cannula. The nurse suspects the patient is experiencing oxygen toxicity after noting which finding?

Restlessness

The nurse administering beclomethasone to a patient can help reduce side effects by instructing the patient to perform which action?

Rinse mouth thoroughly after each use

The nurse is caring for a patient with severe chronic obstructive pulmonary disease (COPD) who has frequent exacerbations. What treatment strategy does the nurse recognize would be most beneficial for this patient?

Roflumilast

While reviewing the laboratory reports of a patient with a chronic cough, dyspnea, and lung inflammation, the nurse finds that the patient has a forced expiratory volume of 55%. Which treatment strategy would be most effective for this patient?

Salmeterol and formotero

The nurse is transporting an oxygen dependent patient to the radiology department. Which mask would be most effective during transportation?

Simple face mask

A patient with chronic obstructive pulmonary disease (COPD) experiences dyspnea and has a forced expiratory volume (FEV1) of 70% of predicted value. The nurse expects that what will be included on the patient's treatment plan?

Since the patient has FEV1 of 70% and is suffering from mild COPD, the patient may benefit from the use of short-acting bronchodilators. Budesonide, being a steroid, is not used for treating mild COPD. Use of theophylline in COPD is controversial and should be used only in patients who do not respond to other drugs. A combination of fluticasone and salmeterol is not required for treating mild COPD; this medication is prescribed to patients who have a FEV1 of less than 60%.

A patient experiences chronic sinusitis. The nurse recognizes that which treatment strategy may help relieve the patient's symptoms caused by impaired ciliary movement in the airway?

Sinusitis is a painful inflammation of the sinuses due to mucus plugging the nasal passages. A patient with sinusitis has impaired ciliary movement due to airway obstruction by the mucus. An airway clearance technique through postural drainage by percussion and vibration or frequent chest wall oscillation helps to relieve symptoms of sinusitis. Dornase alfa helps to degrade deoxyribonucleic acid of neutrophils in cystic fibrosis sputum. β2-adrenergic agonists help with bronchodilation. Huff's coughing method helps to relieve symptoms for the patient with other airway obstructive disorders.

The nurse provides teaching to a patient with asthma who has been advised to use nebulization. What should the nurse include in the instructions about nebulization? Multiple choice question Hold the inspiration for 10 seconds. Breathe rapidly between forced breaths. Sit in an upright position during the treatment. Do not cough after the nebulization treatment.

Sit in an upright position during the treatment. Nebulization involves administering drug solution as mists produced by small machines called nebulizers. An upright position allows for efficient breathing that ensures adequate penetration and deposition of the aerosolized medication. The patient should hold the inspiration for two to three seconds to ensure penetration of the medication. The patient should practice deep breathing in between the forced breathing to prevent alveolar hypoventilation. The patient should be encouraged to cough effectively after the nebulization to mobilize the secretions.

The nurse is teaching a patient how to use a hand-held nebulizer. Which guideline is correct? Multiple choice question Sit in an upright position during the treatment. Take short, shallow breaths while inhaling the medication. Rinse the nebulizer equipment under running water once a week. During the treatment, breathe in and hold the breath for five seconds.

Sit in an upright position during the treatment. The patient is placed in an upright position that allows for most efficient breathing to ensure adequate penetration and deposition of the aerosolized medication. The patient must breathe slowly and deeply through the mouth and hold inspirations for two or three seconds. Deep diaphragmatic breathing helps ensure deposition of the medication. Instruct the patient to breathe normally in between these large forced breaths to prevent alveolar hypoventilation and dizziness. After the treatment instruct the patient to cough effectively. An effective home-cleaning method is to wash the nebulizer equipment daily in soap and water, rinse it with water, and soak it for 20 to 30 minutes in a 1:1 white vinegar-water solution, followed by a water rinse and air drying.

To promote airway clearance in a patient with pneumonia and asthma, the nurse instructs the patient to perform which action? Multiple choice question Perform pursed-lip breathing Wear supplemental oxygen at all times Sit upright while using the flutter device Use the incentive spirometer 10 times per hour

Sit upright while using the flutter device The flutter device is used to increase mucus production to promote airway clearance and gas exchange; it should be used while the patient is in an upright position. Supplemental oxygen may not be indicated depending on the oxygen saturation level. Pursed-lip breathing and the incentive spirometer will not promote airway clearance.

The nurse is caring for a patient with suspected chronic obstructive pulmonary disease (COPD). What diagnostic test should the nurse monitor for confirmation of the presence of COPD?

Spirometry

A patient presents with a productive cough and a body temperature of 102° F. The patient's white blood cell (WBC) count is 15,000/mm3. The nurse expects that what diagnostic test will be prescribed?

Sputum culture test

A patient receives a prescription for omalizumab. What medication route of administration and schedule will the nurse include in the information provided to the patient? Multiple choice question Intravenously, every four to six hours Two to four puffs, every 20 to 30 minutes Subcutaneously, every two to four weeks Orally, one hour before meals or two hours after meals

Subcutaneously, every two to four weeks Omalizumab is a monoclonal antibody that decreases circulating free IgE levels. The patient with asthma will take the medication subcutaneously every two to four weeks. Intravenous corticosteroids are administered every four to six hours. Albuterol nebulization in the patient with asthma is given two to four puffs at a time, every 20 to 30 minutes. Leukotriene modifiers are taken orally, one hour before meals or two hours after meals.

A patient in the emergency ward is dyspneic and speaks unclear words. Assessment findings include a respiratory rate of 45 breaths/minute, a pulse of 130 beats/minute, oxygen saturation of 90%, and neck vein distention. Which treatment does the nurse expect will help alleviate the patient's symptoms?

Supplementary oxygen through nasal cannula

A patient in the emergency ward is dyspneic and speaks unclear words. Assessment findings include a respiratory rate of 45 breaths/minute, a pulse of 130 beats/minute, oxygen saturation of 90%, and neck vein distention. Which treatment does the nurse expect will help alleviate the patient's symptoms? Multiple choice question Administration of ipratropium orally Three puffs of albuterol every 30 minutes Supplementary oxygen through nasal cannula Obtaining peak flow rate and monitoring the patient continuously

Supplementary oxygen through nasal cannula The patient with a severe attack of asthma has an elevated respiratory rate, decreased oxygen saturation and elevated pulse, and the inability to speak, which indicate a severe airway obstruction. The patient may also have neck vein distension. Hence the nurse should anticipate correction of hypoxemia and improve ventilation in the patient with supplementary oxygenation by nasal prongs. This helps to keep oxygen saturation above 90 percent. Administration of ipratropium does not provide additional benefits to the patient. Three puffs of albuterol every 30 minutes help to resolve the symptoms in the patients with mild asthma. Obtaining the peak flow rate and continuous monitoring of the patient is critical during asthma attack, but will not alleviate the patient's symptoms.

An infant is undergoing evaluation for possible cystic fibrosis. The nurse anticipates that which procedure will be prescribed as the main diagnostic test for cystic fibrosis?

Sweat chloride test

Which diagnostic test will help determine the condition of a patient that has a productive cough, perfuse sweating, local hypoxia, and arteriolar vasoconstriction?

Sweat test

The nurse is assigned to care for a patient in the emergency department admitted with an exacerbation of asthma. The patient has received a β2-adrenergic agonists bronchodilator and supplemental oxygen. If the patient's condition does not improve, the nurse should anticipate what as the most likely next step in treatment?

Systemic corticosteroids

The nurse is assigned to care for a patient in the emergency department admitted with an exacerbation of asthma. The patient has received a β 2-adrenergic agonists bronchodilator and supplemental oxygen. If the patient's condition does not improve, the nurse should anticipate what as the most likely next step in treatment? Multiple choice question Biofeedback therapy Intravenous (IV) fluids Systemic corticosteroids Pulmonary function testing

Systemic corticosteroids Systemic corticosteroids speed the resolution of asthma exacerbations and are indicated if the initial response to the β 2-adrenergic agonists bronchodilator is insufficient. IV fluids may be used, but not to improve ventilation. Biofeedback therapy and pulmonary function testing may be used after recovery to assist the patient and monitor the asthma.

The nurse determines that a patient is experiencing the most common adverse effect of albuterol after noting which sign?

Tachycardia

The nurse determines that a patient is experiencing the most common adverse effect of albuterol after noting which sign? Multiple choice question Diarrhea Headache Tachycardia Oral candidiasis

Tachycardia Tachycardia is a common adverse effect of the use of inhaled β 2-adrenergic agonists because of its stimulant effect. Headache, diarrhea, and oral candidiasis are not associated adverse effects of albuterol.

A patient is experiencing an exacerbation of chronic obstructive pulmonary disease (COPD) and requires supplemental oxygen. To deliver the precise amount of oxygen, the nurse should use which type of equipment?

The Venturi mask delivers precise concentrations of oxygen and should be selected whenever this is a priority concern. A non-rebreather and simple face mask are less precise in terms of the amount of oxygen delivered. The laryngeal mask airway is an invasive airway used for surgical procedures or emergency situations.

A four-year-old child with cystic fibrosis (CF) experiences a cough associated with viscous, purulent, and greenish sputum. The nurse expects that which organism will be found in the patient's sputum?

The child has a frequent cough with viscous, purulent, and greenish sputum, which indicates a bacterial infection. The most common bacteria that affect children are Staphylococcus aureus. Burkholderia cepacia is a less common and most serious type of organism that causes CF in children. Haemophilus influenza and Pseudomonas aeruginosa are the organisms that cause CF in adults.

The nurse is teaching a patient pursed-lip breathing (PLB). What instructions by the nurse are correct?

The exhalation period needs to be three times as long as the inhalation period. Patients need to inhale slowly and deeply through the nose first. PLB should be used before, during, and after any activity that causes shortness of breath. While exhaling, the facial muscles need to be relaxed without any puffing out of the cheeks.

The licensed practical nurse is caring for a chronic obstructive pulmonary disease (COPD) patient who has severe hypoxemia. The primary health care provider prescribed oxygen administration for this patient. Which intervention can the nurse perform for the patient?

The licensed practical nurse should adjust the oxygen flow rate depending on the desired oxygen level for patients with COPD who are receiving oxygen therapy. Teaching patients about home oxygen use is not appropriate because this is the responsibility of a registered nurse. If the oxygen level is raised more than the requirement, it will lead to hypoxia. The level of carbon dioxide can be easily decreased, but it is very difficult to reduce the level of oxygen in the body. Therefore, the licensed practical nurse should not provide the patient with a high amount or continuous supply of oxygen.

Which parameter should the nurse consider while assessing a patient with cystic fibrosis (CF)?

The nurse should assess for thick and abundant bronchial mucus due to improper airway clearance in a patient with cystic fibrosis. Recurring lung infections occur due to impaired gas exchange. A decrease in life expectancy is related to ineffective dietary intolerance and weakness. Altered pancreatic enzyme production occurs mainly due to ingestion of nutrients less than the body requires.

Which should the nurse include when providing information related to health promotion and disease prevention to a patient with cystic fibrosis (CF)? Select all that apply.

The nurse should encourage the patient to consume more fluids and caloric supplements to enhance nutritional support in the patient. The nurse should advise the patient to learn about vascular access devices, which may be used for medication administration. Aerobic exercise is effective to clear the patient's airway. The patient must consume more dietary salt to replenish chloride stores in the body. The patient needs support systems; recommending the limitation of relationships is not appropriate.

A patient experiencing severe wheezing arrives in the emergency department and is diagnosed with severe exacerbation of asthma. During the admission assessment, the nurse on the inpatient unit notes that the patient continues to struggle with breathing; however, there is an absence of wheezing. How should the nurse interpret the assessment findings?

The patient has respiratory failure and needs mechanical ventilation

A patient experiencing severe wheezing arrives in the emergency department and is diagnosed with severe exacerbation of asthma. During the admission assessment, the nurse on the inpatient unit notes that the patient continues to struggle with breathing; however, there is an absence of wheezing. How should the nurse interpret the assessment findings? Multiple choice question The patient is hypoxic and needs oxygen therapy. The patient has improved because there is no wheezing. The patient has respiratory failure and needs mechanical ventilation. The patient has retained secretions and needs chest physiotherapy.

The patient has respiratory failure and needs mechanical ventilation. A silent chest or absence of wheezing in a patient who had been having severe wheezing indicates an impending respiratory failure. The patient may need mechanical ventilation to support respiration. It is a sign of severe obstruction and it is a life-threatening condition. It is not a sign of improvement. Oxygen therapy may not help the patient, because there is an obstruction in the airway. Chest physiotherapy helps in removing secretions from the airway, but may not be helpful in patients who are at risk of respiratory failure.

What does the nurse interpret from finding that a patient, after being treated for chronic cough and dyspnea associated with inflammation in lung parenchyma, loses muscle mass?

The patient is on corticosteroid therapy

The nurse is teaching a student nurse about breath sounds in a patient with asthma. Which statement made by the student nurse indicates the need for further teaching?

The patient wheezes loudly during expiration when the asthma is severe

Before discharge, the nurse discusses activity levels with a 61-year-old patient with chronic obstructive pulmonary disease (COPD) and pneumonia. Which exercise goal is most appropriate once the patient is recovered fully from this episode of illness?

The patient will benefit from mild aerobic exercise that does not stress the cardiorespiratory system. The patient should be encouraged to walk for 20 minutes/day, keeping the pulse rate less than 75% to 80% of maximum heart rate (220 - patient's age).

Which complications in a patient with chronic obstructive pulmonary disorder (COPD) require acute intervention? Select all that apply.

The patient with chronic obstructive pulmonary disorder (COPD) may develop complications such as pneumonia, cor pulmonale, and exacerbations of COPD, which require acute interventions. After the crisis is resolved, the patient has to undergo assessment for the degree and severity of the underlying respiratory problem. This information helps the nurse plan a better care plan. Atelectasis and mucoid impact are complications of asthma.

Which laboratory abnormality observed by the nurse confirms the diagnosis of a patient with low body mass index, wheezing, coughing, frequent pneumonia, bleeding, and sweating?

The patient with low body mass index, wheezing, coughing, frequent pneumonia, bleeding, and sweating may have cystic fibrosis ( CF) and impaired movement of sodium and chloride ions in and out of the epithelial cells. The high concentrations of sodium and chloride in the sweat of the patient with CF results from decreased chloride reabsorption in the sweat duct. Abnormal levels of unabsorbed dietary fat lead to steatorrhea. Chronically elevated liver enzymes lead to liver cirrhosis. Elevated inflammatory cytokines may result in osteopenia and osteoporosis.

What would be the appropriate nursing intervention for the patient whose laboratory report shows oxygen saturation of 45 mm Hg and carbon dioxide saturation of 60 mm Hg?

The patient with oxygen saturation of 45 mm Hg and carbon dioxide saturation of 60 mm Hg has severe hypoxemia and hypercapnic respiratory failure. The patient should be advised to plan sexual activity during the daytime, when the patient's breathing is normal. The ideal time would be in late morning or early afternoon before meals. If the patient moved to a high altitude, he or she might suffocate due to low partial pressure of oxygen levels. The patient should walk for 10 to 20 minutes three times per week, breathing through the nose with one step and through a pursed mouth for the next three steps. The patient should be administered a β2-adrenergic agonist five minutes after exercise to provide time for the patient to recover and return to a normal baseline.

When performing pilocarpine iontophoresis in a patient with steatorrhea, the nurse should perform the steps of the procedure in what order?

The pilocarpine iontophoresis is a sweat chloride test. The nurse should provide education to the patient before the procedure. This will allow the patient an opportunity to seek clarification as well as gaining knowledge about what to expect during the procedure. For the procedure, first pilocarpine is placed on the patient to stimulate sweat by a small electric current. The patient may feel tingling and warmth during the treatment; therefore the nurse should make the patient comfortable by communicating while performing the procedure. The sweat is collected on a gauge within five minutes, and the sweat is analyzed for chloride levels. The chloride level in the patient with steatorrhea will be above 60 mmol/L. The test is performed simultaneously for the other hand to confirm the diagnosis.

Which finding helps to diagnose cystic fibrosis in a newborn?

The presence of meconium ileus helps to diagnose 20 percent of cases of cystic fibrosis (CF) in newborns. Gallstones and liver cirrhosis occur in some patients with CF. Liver cirrhosis occurs over time due to an accumulation of mucus in the liver of patients with cystic fibrosis. Terminal ileus does not indicate the presence of CF in the newborn.

A nurse is caring for a patient diagnosed with chronic obstructive pulmonary disease (COPD). The lab reports of the patient reveal a hemoglobin level of 20 g/dL. What could be the reason for the increased hemoglobin?

The production of red blood cells increases in response to hypoxia

When teaching the patient with bronchiectasis about manifestations to report to the health care provider, which manifestation should be included?

The significant clinical manifestations to report to the health care provider include increasing dyspnea, fever, chills, increased sputum production, bloody sputum, and chest pain. Although drinking at least 3 L of low sodium fluid will help liquefy secretions to make them easier to expectorate, the health care provider does not need to be notified if the patient cannot do this

A patient with chronic obstructive pulmonary disease (COPD) is advised to use oxygen therapy at home. A nurse provides discharge instructions about how to prevent respiratory infection. What should be included in the teaching? Select all that apply.

The strategies to reduce infection while using oxygen therapy at home include changing the cannula every two weeks, washing the cannula twice a week with liquid soap, and using a mouthwash several times a day. The nasal cannula may become contaminated with repeated use and should be changed every week. It should also be cleaned twice a week to remove the particulate material and moisture. Frequent use of mouthwash helps to keep the oral cavity clean and prevent infection. Removing the secretions that are coughed out reduces the risk of infection. The oxygen concentrator cabinet should be cleaned every day, not weekly.

The nurse is caring for a child who is suspected of having cystic fibrosis (CF). The nurse knows that the gold standard of diagnosing this condition is:

The sweat chloride test is the most definitive test in children, but may not be conclusive in adults. Patients with CF secrete four times the normal amounts of sodium and chloride in their sweat. Genetic tests may be used if the sweat test is inconclusive. A lung biopsy is not used to diagnose this condition. Pancreatic insufficiency is related to CF but does not specifically diagnose the disease.

A patient with a forced expiratory volume of 80 percent and a three-day history of breathlessness develops nausea, vomiting, headache, tachycardia, and dysrhythmias. The nurse suspects that which medication that the patient takes is the cause of the patient's symptoms?

Theophylline

A patient with a forced expiratory volume of 80 percent and a three-day history of breathlessness develops nausea, vomiting, headache, tachycardia, and dysrhythmias. The nurse suspects that which medication that the patient takes is the cause of the patient's symptoms? Multiple choice question Albuterol Omalizumab Mometasone Theophylline

Theophylline A patient with forced expiratory volume of 80 percent and breathlessness for three days has mild asthma. The patient may develop nausea, vomiting, headache, tachycardia, and dysrhythmias upon theophylline use. Theophylline is a methylxanthine bronchodilator with narrow therapeutic use. Therefore its use is very limited. The patient's serum concentration levels should be monitored regularly to determine that the drug is within therapeutic window. Albuterol is a short-acting beta agonist that is safe in the patient with mild asthma. Mometasone is a corticosteroid that may cause local irritation like cough and hoarseness upon inhalation. Omalizumab is a monoclonal antibody that decreases circulating free IgE levels in the patient with allergic asthma.

An asthmatic patient is in acute respiratory distress. The nurse auscultates the lungs and notes cessation of inspiratory wheezing. How does the nurse interpret this finding?

There is worsening airway inflammation and bronchoconstriction

The nurse recognizes the patient with which reported awakening as mildly asthmatic?

Three to four days per month

The nurse recognizes that which intervention will be beneficial to the patient with pleuritic chest pain, dyspnea, wheezing, and mouth bleeding?

To prevent infection, encourage the patient to receive the pneumococcal and influenza vaccinations

A patient has had chronic obstructive pulmonary disease (COPD) for three months. Further assessment reveals that forced expiratory volume in one second (FEV1) is 65 percent. Which treatment option would be most appropriate for this patient?

Treatment with albuterol and ipratropium combination

The nurse is caring for an adult patient with bronchiectasis. The nurse knows that the primary cause of this disease in adults is related to what?

Untreated or delayed treatment of bacterial lung infections

The nurse is teaching energy conservation techniques to a patient with severe dyspnea and oxygen saturation of 50 mm Hg. Which action of the patient indicates effective learning?

Using a tripod posture and placing the mirror on the table while using an electric razor

Which intervention by the student nurse indicates the need for further teaching while performing percussion of a patient with chronic obstructive pulmonary disorder (COPD)?

Using both hands at once during percussion

A patient with an acute exacerbation of chronic obstructive pulmonary disease (COPD) needs to receive precise amounts of oxygen. Which equipment should the nurse prepare to use?

Venturi mask

The nurse determines that the patient has understood medication instructions about the use of a metered dose inhaler (MDI) when the patient performs which action?

Waits one minute between each puff from the MDI

The nurse determines that the patient has understood medication instructions about the use of a metered dose inhaler (MDI) when the patient performs which action? Multiple choice question Inhales rapidly when activating the inhaler Holds the MDI sideways to increase ease of use Waits one minute between each puff from the MDI Breathes through the nose with activation of the MDI

Waits one minute between each puff from the MDI The patient should wait at least one minute in between puffs to increase medication dispersion throughout the lungs. The patient should inhale slowly, hold the MDI upright, and breathe through the mouth.

What discharge instructions should the nurse include for the patient with asthma? Select all that apply. Multiple selection question Wash the nebulizer regularly. Wash the mouth after taking albuterol. Medication noncompliance may lead to exacerbations. It is important to know the purpose and side effects of prescribed medications. Wash respiratory equipment with a solution of one-part water to two parts white vinegar.

Wash the nebulizer regularly. Medication noncompliance may lead to exacerbations. It is important to know the purpose and side effects of prescribed medications. The nurse should advise the patient to soak the nebulizer in soap and water as the hot water kills the germs on it. Educating the patient about drugs and devices before discharge from the hospital is the responsibility of the nurse. The nurse should warn the patient about the dangers of noncompliance with the medications to avoid triggering asthma attacks. The patient should wash the mouth after inhalation of corticosteroids. The patient must wash respiratory equipment with 1:1 water and white vinegar.

The nurse is caring for a patient with an acute exacerbation of asthma. Following initial treatment, what finding indicates to the nurse that the patient's respiratory status is improving? Multiple choice question Wheezing becomes louder. Cough remains nonproductive. Vesicular breath sounds decrease. Aerosol bronchodilators stimulate coughing.

Wheezing becomes louder. The primary problem during an exacerbation of asthma is narrowing of the airway and subsequent diminished air exchange. As the airways begin to dilate, wheezing gets louder because of better air exchange. After a severe asthma exacerbation, the cough may be productive and stringy. Vesicular breath sounds will increase with improved respiratory status. Coughing after aerosol bronchodilators may indicate a problem with the inhaler or its use.

The nurse provides information to a group of nursing students about women with cystic fibrosis (CF) regarding conception and pregnancy. Which statement made by a student nurse related to female patients with CF indicates understanding of the education?

Women with CF may have menstrual irregularities and secondary amenorrhea during exacerbations. The female CF patient has the ability to breastfeed. The woman with cystic fibrosis has a thick cervical mucus membrane and is capable of becoming pregnant. There is no need for assisted reproductive techniques for the woman to conceive.

A 61-year-old patient with asthma is admitted to the hospital. The nurse understands that symptoms of asthma include what? Select all that apply.

cough, wheeze, chest tightness

The nurse recalls that which type of asthma involves an acute asthma attack triggered by an upper respiratory infection?

intrensic

The nurse is assigned to care for an 83-year-old patient with an acute asthma exacerbation. Which arterial blood gas (ABG) result would prompt the nurse to notify the provider immediately?

pH 7.30, PaO2 74 mm Hg, PaCO2 65 mm Hg

The nurse is collecting data on four patients with a history of mild asthma. Which patient is most likely to experience wheezing, congestion and angioedema?

patient C

The nurse identifies the nursing diagnosis of activity intolerance for a patient with asthma. In patients with asthma, the nurse assesses for which etiologic factor for this nursing diagnosis?

work of breathing

Which treatment may increase restlessness and insomnia in a patient with chronic obstructive pulmonary disease (COPD)?

β2 agonists may cause restlessness and insomnia in patients with COPD. Anticholinergics are not associated with insomnia. Massage and postural drainage techniques will not lead to insomnia and restlessness. Oxygen supplementation through a nasal mask will not cause restlessness and insomnia.

What complication does the nurse expect in a child with chronic pulmonary disease who is diagnosed with α1-antitrypsin (AAT) deficiency?

AAT deficiency is an autosomal recessive disorder associated with mutations in the SERPINA1 gene, S and Z alleles. These mutations result in abnormalities of the lungs and liver. The mutation in the SERPINA1 gene does not affect the functions of the kidneys, intestine, or urinary tract.

A patient presents with acute exacerbation of asthma. The nurse expects which strategies will be included in the treatment plan? Select all that apply.

Acute exacerbation of asthma may be life-threatening and needs immediate intervention. Administering 100% oxygen helps to relieve hypoxia and improve tissue oxygenation. Nebulization with SABA helps to relax the airways and promote airflow. Corticosteroids are administered to blunt the hyperactive immune response. Sedatives should be avoided as they may depress the respiratory center and worsen dyspnea. Antibiotics are not administered unless there are symptoms of pneumonia.

The patient has a prescription for each of the inhalers. Which one should the nurse offer to the patient at the onset of an asthma attack?

Albuterol is a short-acting bronchodilator that should be given initially when the patient experiences an asthma attack. Salmeterol is a long-acting β2-adrenergic agonist, which is not used for acute asthma attacks. Beclomethasone is a corticosteroid inhaler and is not recommended for an acute asthma attack. Ipratropium bromide is an anticholinergic agent that is less effective than β2-adrenergic agonists. It may be used in an emergency with a patient unable to tolerate short-acting β2-adrenergic agonists (SABAs).

The nurse is assisting a patient to learn self-administration of beclomethasone, two puffs inhaled every six hours. What should the nurse explain as the best way to prevent oral infection while taking this medication?

Because beclomethasone is a corticosteroid, the patient should rinse the mouth with water following the second puff of medication to reduce the risk of fungal overgrowth and oral infection. The mouth should be rinsed after the second puff, not before each puff. Hard candy or breath mints will not prevent oral infection.

A patient experiences exacerbations of wheezing and breathlessness when exercising in the early morning. The nurse learns that patient has a three-year history of sinusitis. What is appropriate to be included on the patient's plan of care?

Both acute and chronic sinusitis make asthma worse, causing severe inflammation of the mucous membranes. Such patients should undergo removal of large nasal polyps to help control exacerbations of asthma. Beta-antagonists promote bronchoconstriction, leading to further airway obstruction and breathlessness in the patient. The nurse should advise the patient to avoid exercise when the air is cool and dry to prevent exacerbations. Intravenous corticosteroids are as effective as inhaled corticosteroids. However, the patient may experience exacerbations upon administration of corticosteroids intravenously. This intervention may increase fluid accumulation in the sinuses, further leading to pain.

A patient in the outpatient clinic has symptoms including chronic cough, sputum production, and dyspnea. On taking a detailed history of the patient, the nurse finds that this patient has a prolonged exposure to smoke. Which condition would the nurse most likely suspect the patient to have?

COPD symptoms include cough, sputum production, and dyspnea. In addition, this patient has a history of exposure to allergens such as smoke. Tuberculosis is a bacterial infection with a low-grade fever and weight loss. Dyspnea is a late symptom of tuberculosis. Pneumonia is an infection with a cough, dyspnea, fever, chills, and pleuritic chest pain. Influenza is a viral infection with sneezing, watery eyes and nose, and fever.

A patient experiencing an acute asthma exacerbation has received a bronchodilator and supplemental oxygen. Which treatment should the nurse anticipate if the patient's condition remains unchanged?

Corticosteroids are antiinflammatories that are effective in treating respiratory distress caused by bronchoconstriction. The patient would be given this medication as an IV push medication. Chest x-ray is not a treatment of an asthma exacerbation. IV antibiotics are not indicated in the absence of infection. Peak flow measurements can measure airflow, but will not improve the patient's condition.

A patient with asthma experiences anaphylaxis. Which medication should the nurse prepare to administer?

Epinephrine helps to resolve anaphylactic reactions in the patient with asthma. Administer epinephrine either subcutaneously or intramuscularly to treat the patient. The nurse should monitor the blood pressure and electrocardiogram of the patient closely after administration of the drug. Timolol is a beta-blocker that may trigger the symptoms of asthma in the patient. Magnesium sulfate helps to treat the patient with severe or life threatening asthma. Sodium bicarbonate helps to treat severe metabolic or respiratory acidosis.

A nurse is caring for a patient diagnosed with chronic obstructive pulmonary disease (COPD). The lab reports of the patient reveal a hemoglobin level of 20 g/dL. What could be the reason for the increased hemoglobin?

In COPD, there is chronic hypoxia. To compensate for it, the production of RBC increases, leading to polycythemia or increased hemoglobin levels. The patient cannot have a hemoglobin level of 20 g/dL by eating iron-rich food. Patients with COPD usually have compromised heart function. The patient does have COPD and polycythemia is a defense response of the body against hypoxemia.

The nurse is assessing the respiratory system of an 87-year-old patient admitted with emphysema. Which assessment findings does the nurse expect? Select all that apply.

In patients with emphysema, the nurse would hear decreased breath sounds, barrel chest, and wheezes. Crackles would indicate fluid not consistent with emphysema and increased tactile fremitus, and dullness would indicate a consolidation of fluid or tissue in the lungs.

The nurse is overseeing an exercise program for patients with mild chronic obstructive pulmonary disease (COPD). Part of the program involves walking. Vital signs are taken after walking. The nurse becomes concerned when a 60-year-old patient's pulse rate is what?

Parameters that may be monitored in the patient with mild COPD are resting pulse and pulse rate after walking. Pulse rate after walking should not exceed 75% to 80% of the maximum heart rate (maximum heart rate is age in years subtracted from 220). 220 - age (60) = 160 (maximum heart rate). 120 is 75% of the patient's maximum heart rate.

When teaching a patient with asthma about ways to reduce the severity of asthma and asthma attacks, which measures should be included? Select all that apply.

Patients with asthma should be taught to avoid food irritants and animals with fur. Identifying personal triggers can help to avoid them. Going out in the cold air and the use of NSAIDs are not recommended, because they can precipitate an asthma attack.

A patient needs to receive oxygen at a 28% concentration. The nurse will set up which oxygen delivery device?

The Venturi mask is a high-flow device that delivers fixed concentrations of oxygen (e.g., 24%, 28%), independent of the patient's respiratory pattern. Nasal cannulas, simple face masks, and non-rebreather masks are examples of low-flow devices that deliver less precise concentrations of oxygen.

Infection can be a major hazard of O2 administration. Heated nebulizers present the highest risk. Which is the most common organism found?

The constant use of humidity supports bacterial growth, with the most common organism being P. aeruginosa. Rickettsia prowazekii, Clostridium perfringens, and Bordatella pertussis are not the most common organisms found in this case.

The nurse is assigned to care for a patient who has anxiety and an exacerbation of asthma. What is the primary reason for the nurse to carefully inspect the chest wall of this patient?

The nurse physically inspects the chest wall to evaluate the use of intercostal (accessory) muscles, which gives an indication of the degree of respiratory distress experienced by the patient. Allowing time to calm the patient, observing for diaphoresis, and monitoring for bilateral chest expansion are correct, but they are not the primary reason for inspecting the chest wall of this patient.

What discharge instructions should the nurse include for the patient with asthma? Select all that apply.

The nurse should advise the patient to soak the nebulizer in soap and water as the hot water kills the germs on it. Educating the patient about drugs and devices before discharge from the hospital is the responsibility of the nurse. The nurse should warn the patient about the dangers of noncompliance with the medications to avoid triggering asthma attacks. The patient should wash the mouth after inhalation of corticosteroids. The patient must wash respiratory equipment with 1:1 water and white vinegar.

Which complication does the nurse monitor in the patient who is using an oxygen-conserving cannula?

The oxygen-conserving cannula is applied as a moustache or pendant type, over the ears. The patient may experience necrosis over the tops of the ears due to constant contact. The oxygen-conserving cannula has pipes placed directly over the naris, so the loss of oxygen into atmosphere is very little. A patient who is using a tracheostomy collar will have tissue damage at the tracheostomy tube. While using non-rebreather masks, a decrease in the fraction of inspired oxygen, or FIO2, may occur.

The nurse is caring for a patient with an acute exacerbation of asthma. Following initial treatment, what finding indicates to the nurse that the patient's respiratory status is improving?

The primary problem during an exacerbation of asthma is narrowing of the airway and subsequent diminished air exchange. As the airways begin to dilate, wheezing gets louder because of better air exchange. After a severe asthma exacerbation, the cough may be productive and stringy. Vesicular breath sounds will increase with improved respiratory status. Coughing after aerosol bronchodilators may indicate a problem with the inhaler or its use.

After the inhalation of puffs of mometasone, a patient develops oropharyngeal candidiasis, hoarseness and dry cough. What action should the nurse take to reduce the symptoms?

Upon inhalation into the pharynx, mometasone may cause local irritation such as oropharyngeal candidiasis, hoarseness, and dry cough. Hence the patient should rinse the mouth either with water or with mouthwash after inhalation. The patient may not be benefit by pausing between the puffs. Asking the patient to use a spacer or holding device for inhalation of corticosteroids can be helpful in getting more medication into the lungs. However, it does not reduce the symptoms of candidiasis. The next dose is given to the patient only upon further advice from the practitioner.

A nurse observes a patient using a dry powder inhaler device. The nurse should correct which patient actions? Select all that apply.

When using a dry powder inhaler, the patient should not breathe into the inhaler, because this affects the dosing. Inhaling more than one puff with each inspiration may cause waste of the medication. The patient should not shake the medicine before using it because it can alter the dosing. Deep and quick breathing is the proper technique, because it ensures that the medicine moves deep into the lungs. The patient should be encouraged to hold the breath beyond 10 seconds to help in penetration of the dry powder.

The nurse reviews pursed-lip breathing with a 61-year-old patient with emphysema. Which statement made by the patient indicates correct understanding of this technique?

The focus of pursed-lip breathing is to slow down the exhalation phase of respiration, which decreases bronchial collapse and subsequent air trapping in the lungs during exhalation. Exhalation should be two to three times longer than inhalation. Lying down is not indicated, lips should be pursed during the exhalation phase, and it is not necessary to puff out the cheeks at any time during this technique

What instruction should the nurse give to a patient with asthma who has received a prescription for albuterol and ipratropium nebulization?

The nurse should instruct the patient to cough effectively after the treatment to prevent hypoventilation and local irritation in the throat. This also helps to disperse accumulated drug in the airway. The nurse should advise the patient to sit upright to ensure efficient breathing and adequate penetration of the aerosol. The patient on corticosteroid inhalation needs to clean the mouth by performing mouthwash to avoid local irritation in the throat. While the patient is on aerosolized medication, the nurse should instruct the patient to breathe slowly and deeply.

A nurse is caring for a patient who is having an acute asthma attack. Which interventions should the nurse question? Select all that apply.

The nurse should question prednisone 10 mg, PO because this is an urgent situation that requires intravenous, intramuscular, and inhaled medications, rather than oral ones. There is not enough documentation to indicate that this patient requires administration of blood products, and blood products are not used to treat oxygen saturation. Peak flow readings, arterial blood gases, and continuous oxygen would be considered the standard of care for this patient.

An adolescent who has a history of asthma experiences wheezing after vigorous exercise. What nursing instruction is helpful for this patient?

The patient is suffering from exercise-induced asthma. The nurse should encourage the patient to avoid exposure to dry air, because it precipitates exercise-induced asthma. Exposure to cold air also precipitates asthma. The patient should be discouraged from performing vigorous exercise, because it can precipitate asthma. Swimming in indoor heated pools should be encouraged over swimming outdoors, because outdoor swimming can trigger asthma attacks.

The health care provider has prescribed salmeterol for a patient with asthma. In reviewing the use of dry powder inhalers (DPIs) with the patient, what instructions should the nurse provide?

The patient should be instructed to tightly close the lips around the mouthpiece and breathe in deeply and quickly to ensure the medicine moves down deeply into the lungs. Dry powder inhalers do not require spacer devices. The patient may not taste or sense the medicine going into the lungs. The inhaler should not be placed several inches in front of the mouth.

The nurse identifies the nursing diagnosis of activity intolerance for a patient with asthma. In patients with asthma, the nurse assesses for which etiologic factor for this nursing diagnosis?

When the patient does not have sufficient gas exchange to engage in activity, the etiologic factor is often the work of breathing. When patients with asthma do not have effective respirations, they use all available energy to breathe and have little left over for purposeful activity. Fear of suffocation, effects of medications or anxiety, and restlessness are not etiologies for activity intolerance for a patient with asthma.

An asthmatic patient was prescribed theophylline. A nurse understands that the patient is at risk for tachycardia and seizures. In regard to safety, the nurse expects that what will be included on the patient's treatment plan?

Tachycardia and seizures are known toxic effects of theophylline at higher blood levels. In addition, the drug has a narrow margin of safety. Therefore monitoring blood levels of theophylline helps to reduce such toxic effects. Caffeine increases the toxic effects of theophylline. Monitoring plasma levels of adrenaline may not help, because tachycardia is not associated with adrenaline in this case. Prophylactic use of diazepam may not prevent seizures that are caused by the use of theophylline.

When admitting a patient with a diagnosis of asthma exacerbation, the nurse will assess for what potential triggers? Select all that apply.

Although the exact mechanism of asthma is unknown, there are several triggers that may precipitate an attack. These include allergens, exercise, air pollutants, upper respiratory infections, drug and food additives, psychologic factors, and gastroesophageal reflux disease (GERD). Decreased humidity is not a trigger.

In which position is it most appropriate for the nurse to place a patient experiencing an asthma exacerbation?

The patient experiencing an asthma attack should be placed in high-Fowler's position and may need to lean forward to allow for optimal chest expansion and enlist the aid of gravity during inspiration. The supine, lithotomy, and reverse Trendelenburg positions will not facilitate ventilation.

A patient is prescribed albuterol with a metered dose inhaler (MDI). After taking the cap off and shaking the inhaler, the nurse should instruct the patient to perform the steps of taking the medication in what order?

After taking the cap off the medicine and shaking the inhaler, the patient breathes out all the air from the lungs and holds the inhaler as recommended. Then, the patient should start breathing in slowly through the mouth by pressing down the inhaler. With five seconds, the patient should start breathing slowly and deeply. The patient should hold the breath for 10 seconds or more as much as possible. This should be repeated three to four times for better relief. The patient has to wait about one minute between puffs to avoid local irritation caused by the drug

A patient with asthma is prescribed ipratropium bromide. The nurse recognizes that the patient may develop what side effect of the medication?

A patient who is taking ipratropium bromide may develop dry mouth due to inhibition of parasympathetic nervous system. Anxiety is a side effect in the patient who is taking an oral medication of a beta-adrenergic agonist. Insomnia is the common side effect of methylxanthine. Corticosteroids upon inhalation may produce local irritation, such as dry cough.

The nurse is caring for a patient who is being mechanically ventilated that has a PaCO2 of 60 mm Hg, a respiratory pH of 6.8, and a PaO2 of 60 mm Hg. Which intervention will benefit the patient?

A patient with PaCO2 of 60 mm Hg and respiratory pH of 6.8 has severe respiratory acidosis. Therefore such a patient must take sodium bicarbonate to treat extreme acidosis. Bronchodilation by a beta-agonist like albuterol is not possible in the patient with extreme acidosis. Bronchoscopy helps to relieve acute attacks by removing thick mucus plugs. Chest physiotherapy is generally not recommended for asthma because it is too stressful for breathless patients.

A patient with asthma is admitted in the emergency department with severe dyspnea and is unable to speak. The nurse finds that the patient looks drowsy and confused. Which other finding does the nurse expect?

A patient with a severe asthma exacerbation is dyspneic at rest and has difficulty speaking. The patient may be unusually drowsy and confused as the arterial blood gas levels deteriorate. Upon auscultation, the nurse would hear no wheezing sound in the patient with life-threatening asthma. Peak flow in the patient reduces to 25 percent of the personal best, due to limited airflow in airway. The patient with reduction in peak flow of air experiences bradycardia with respiratory arrest.

A patient is having an asthma attack, and is short of breath and appears frightened. The nurse understands that possible triggers for asthma exacerbations include which factors? Select all that apply.

Alcohol, GERD, animal dander, perfumes, and cold weather (not humid) are all possible triggers for acute asthma exacerbations.

A patient with dyspnea and hypoxemia has received an initial nebulized short-acting β2-adrenergic agonist (SABA) with ipratropium treatment. The patient's forced expiratory volume in one second is 60 percent, and the peak flow is less than 25 percent of personal best. The nurse anticipates that which medication will be administered?

A patient with dyspnea and hypoxemia and who is on initial treatment with ipratropium has severe asthma. Such a patient has low forced expiratory volume in one second of 60 percent and peak flow less than 25 percent of personal best. Intravenous administration of magnesium sulfate helps to resolve the patient's condition faster. Administration of epinephrine is not indicated for the treatment of asthma exacerbations. However, a subcutaneous or intramuscular injection may be used for acute treatment of anaphylaxis. Administration of theophylline is no longer recommended for asthma exacerbations. Sodium bicarbonate administration is limited to the treatment of severe metabolic or respiratory acidosis.

A patient with a forced expiratory volume of 80 percent and a three-day history of breathlessness develops nausea, vomiting, headache, tachycardia, and dysrhythmias. The nurse suspects that which medication that the patient takes is the cause of the patient's symptoms?

A patient with forced expiratory volume of 80 percent and breathlessness for three days has mild asthma. The patient may develop nausea, vomiting, headache, tachycardia, and dysrhythmias upon theophylline use. Theophylline is a methylxanthine bronchodilator with narrow therapeutic use. Therefore its use is very limited. The patient's serum concentration levels should be monitored regularly to determine that the drug is within therapeutic window. Albuterol is a short-acting beta agonist that is safe in the patient with mild asthma. Mometasone is a corticosteroid that may cause local irritation like cough and hoarseness upon inhalation. Omalizumab is a monoclonal antibody that decreases circulating free IgE levels in the patient with allergic asthma.

A patient experiencing severe wheezing arrives in the emergency department and is diagnosed with severe exacerbation of asthma. During the admission assessment, the nurse on the inpatient unit notes that the patient continues to struggle with breathing; however, there is an absence of wheezing. How should the nurse interpret the assessment findings?

A silent chest or absence of wheezing in a patient who had been having severe wheezing indicates an impending respiratory failure. The patient may need mechanical ventilation to support respiration. It is a sign of severe obstruction and it is a life-threatening condition. It is not a sign of improvement. Oxygen therapy may not help the patient, because there is an obstruction in the airway. Chest physiotherapy helps in removing secretions from the airway, but may not be helpful in patients who are at risk of respiratory failure.

The nurse is transporting an oxygen dependent patient to the radiology department. Which mask would be most effective during transportation?

A simple face mask covers the nose and mouth of the patient and is useful in administering oxygen for very short periods, such as during transportation. A patient who has undergone tracheostomy surgery will have less difficulty breathing with a tracheostomy collar. Oxygen-conserving cannulas help to administer oxygen for long-term therapy. Partial and non-rebreather masks are useful in administering high concentrations of oxygen for short-term periods, or about 24 hours.

A patient presents to the emergency department with sudden-onset wheezing and coughing with progressive respiratory distress. What condition or diagnosis does the nurse recognize?

A sudden onset of coughing and wheezing are the initial and most obvious symptoms of an acute asthma attack. An asthma attack may begin mildly but progress to respiratory distress and arrest if it goes untreated. Acute bronchitis is irritation and inflammation of the mucous-membrane lining of the respiratory tract, usually caused by an infectious agent. Pulmonary edema is fluid accumulation in the lungs due to heart failure or lung injury. The fluid collection impairs gas exchange and may result in respiratory failure. Congestive heart failure, or heart failure, is a condition in which the heart cannot pump effectively. Fluid may accumulate in the lungs. Edema may develop in the lower extremities, and shortness of breath may also occur with increasing frequency and severity.

The nurse is assessing a patient who is having an acute asthma attack. Which nursing intervention is the priority for this patient?

An inhaled short-acting beta2-adrenergic agonist like albuterol is the treatment of choice for providing quick relief to a patient having an acute asthma attack. Humidification is more typically used for chronic obstructive pulmonary disease treatment. Oral corticosteroids are longer-acting and may be used in the long-term management of asthma. The nurse may place the patient in high-Fowler's position, but administering albuterol comes first.

The patient is receiving 3 L of oxygen (O2) via nasal cannula. Which action by the nurse is most appropriate? Select all that apply.

Assessing the bubble-through humidifier if humidity is used, assuring that the patient is wearing the nasal cannula correctly, and adjusting humidification according to patient comfort are correct because oxygen (O2) obtained from cylinders or wall systems is dry. Dry O2 has an irritating effect on mucous membranes and dries secretions. A common device used for humidification when the patient has a cannula or a mask is a bubble-through humidifier. It is important for the nurse to assess the bubble-through humidifier if humidity is used to make sure the humidification is on. This adds to the comfort of the patient. The nurse assesses the patient to make sure the nasal cannula is worn correctly for optimal effect. The cannula can become easily dislodged. Humidification is adjusted according to the patient's comfort level. When oxygen levels are 1 to 4 L, the use of humidification may not be the preference of all patients. Believing that humidification is never needed is incorrect because the use of humidification is a patient preference. The nurse should assess the patient's nose for dryness, not the eyes.

The nurse finds that a patient is experiencing breathlessness, chest tightness, and a cough after climbing stairs. What is the most likely reason for these symptoms?

Asthma that is induced or exacerbated during physical exertion is called exercise-induced asthma (EIA). Typically, EIA occurs after vigorous exercise, not during it. Airway obstruction may occur due to changes in the airway mucosa caused by hyperventilation during exercise, with either cooling or rewarming of air and capillary leakage in the airway wall. Edema of airway walls occurs during respiratory tract infections. A change in responsiveness of the airway occurs on exposure to irritants, as seen with occupational asthma. Patients who are genetically predisposed to develop an allergic (immunoglobulin E-mediated) response have an increased risk of allergic asthma, but it would not cause an exacerbation after climbing stairs.

The nurse is preparing a patient for a procedure with the thoracoscope to reduce the exacerbations of chronic obstructive pulmonary disease (COPD). What procedure should the nurse be sure the patient understands?

Bullectomy is a surgical procedure to reduce the exacerbations of COPD. The patient with COPD may have large bullae, which are resected through a thoracoscope. Airway bypass is a bronchoscopic procedure that helps to treat COPD by creating extra anatomic openings between the diseased lung and bronchi; however, this procedure does not require a thoracoscope. Lung transplantation is effective treatment for the patients with advanced COPD; however, this procedure is not associated with the use of a thoracoscope. Lung volume reduction surgery is a surgical procedure to reduce the size of the lung by removing diseased tissue. It requires a bronchoscope, not a thoracoscope.

A patient with chronic obstructive pulmonary disease (COPD) is experiencing anxiety. What medication as ordered should the nurse administer to this patient?

Buspirone is an anxiolytic psychotropic drug that helps to reduce anxiety in the patient during COPD treatment. Tiotropium and indacaterol are long-acting anticholinergic drugs, which help to treat COPD by reducing inflammation. Indacaterol inhalation is used to control wheezing, shortness of breath, coughing, and chest tightness caused by COPD. Roflumilast is an antiinflammatory agent, which helps to treat the exacerbations of COPD but does not treat anxiety.

The nurse is caring for the patient with chronic obstructive pulmonary disease (COPD). The nurse will include in the patient plan of care:

COPD patients should rest for at least 30 minutes before eating to decrease dyspnea and conserve energy. Encouraging the patient to perform mild exercises 60 minutes before eating, telling the patient to avoid taking bronchodilators before meals, and teaching the patient to discontinue oxygen while eating are incorrect because patients should be encouraged to rest before eating to conserve energy and decrease dyspnea, and the exercises before meals would do the exact opposite. It is okay for the patient to use a bronchodilator before meals if prescribed, because it will decrease dyspnea and conserve energy. If a patient has O2 therapy prescribed, use of supplemental O2 by nasal cannula while eating may be beneficial, because this will decrease dyspnea and conserve energy.

The nurse is caring for a patient with chronic obstructive pulmonary disease. About what medications will the nurse educate the patient that have the side effects of a feeling of early satiety and a bloating sensation? Select all that apply

Chronic cough, dyspnea, and chronic inflammation in lung parenchyma indicate that the patient has chronic obstructive pulmonary disease ( COPD). Theophylline is a methylxanthine drug that relaxes smooth muscles and helps to treat COPD effectively, but it causes gastrointestinal side effects. Corticosteroids help to control swelling in the lungs, although they can also cause abdominal distension, so the patient may experience bloating and feelings of satiety. Diuretics are helpful for treating cor pulmonale, a complication associated with COPD, but they are not associated with feelings of satiety or bloating. Roflumilast is an antiinflammatory medication that helps to reduce inflammation in the lungs but does not cause a bloating sensation or a feeling of satiety. Bronchodilators are not associated with feelings of satiety or bloating.

The nurse is educating a patient with chronic obstructive pulmonary disease (COPD) who continues to smoke cigarettes despite the diagnosis. What complication of smoking should the nurse discuss with the patient?

Chronic smokers develop COPD and are more prone to cardiac complications, because smoking directly affects the function of the lungs and heart. The patient with COPD may experience a loss of appetite due to dyspnea, which may lead to the development of cachexia. Osteoporosis is a complication of COPD that is associated with continuous systemic inflammation. Dyspnea and a loss of appetite are associated with COPD and may manifest as a metabolic syndrome. Cachexia, osteoporosis, and metabolic syndrome are not associated with smoking.

Which corticosteroid can be administered directly through inhalation?

Ciclesonide has reduced local side effects like oropharyngeal candidiasis, hoarseness, and dry cough because it activates the lungs and is administered directly through inhalation. Drugs such as fluticasone, budesonide, and mometasone cause local irritation as they are activated in the pharynx. Therefore these medications require a spacer for delivery into the lungs.

Nursing assessment findings of jugular venous distention and pedal edema would be indicative of what complication of chronic obstructive pulmonary disease (COPD)?

Cor pulmonale is a right-sided heart failure caused by resistance to right ventricular outflow resulting from lung disease. With failure of the right ventricle, the blood emptying into the right atrium and ventricle would be slowed, leading to jugular venous distention and pedal edema.

The nurse caring for a patient with diabetes mellitus and chronic obstructive pulmonary disease (COPD) on oral prednisone will monitor which parameter regularly?

Corticosteroids such as prednisone can lead to elevated blood sugar, especially for patients with diabetes. For this reason, it is useful to monitor the patient's blood sugar. The patient's blood pressure and bowel sounds will not be affected. The hemoglobin A1c will demonstrate average blood sugars over the past three months, which would not evaluate blood sugar since beginning prednisone

The nurse, who has administered a first dose of oral prednisone to a patient with asthma, writes on the care plan to begin monitoring for which patient parameters?

Corticosteroids such as prednisone can lead to weight gain. For this reason, it is important to monitor the patient's daily weight. The drug should not affect the apical pulse, bowel sounds, or deep tendon reflexes.

A child with a nonproductive cough states, "I am having trouble breathing." What action should the nurse take to reduce the severity of breathlessness in the child?

Cough may be the only symptom in patients with cough-variant asthma. The nurse should instruct the child with asthma to sit upright or slightly bent forward, because these positions would help the child to use accessory muscles for respiration. The child should not lie in a supine position, which may increase bronchial tone and cause irritation with stimulation of cough receptors. The child must not walk for half an hour, because it may trigger the symptoms. The nurse should instruct the child to take deep breaths.

The nurse is educating a patient regarding breathing techniques. What technique should the nurse avoid in a patient with chronic obstructive pulmonary disease (COPD) who has marked hyperinflation?

Diaphragmatic breathing is a technique in which the patient breathes with the diaphragm but does not use accessory muscles. The use of the diaphragm increases the work of breathing, so the patient with COPD may have difficulty in breathing. Yoga involves slow, deep breathing. It is helpful for the patient with COPD to perform yoga. Huff coughing is an effective coughing technique that helps to clear sputum from the airways. Pursed-lip breathing is a prolonged exhalation breathing exercise in which the patient exhales for a prolonged time.

The nurse is assessing a patient with asthma who has been diagnosed with a severe and life-threatening exacerbation. What findings would the nurse find?

Early in exacerbation the CO2 level is decreased, but increases if exacerbation is prolonged or severe. With an asthma exacerbation, patients are unable to speak in sentences and only speak a few words at a time before taking a breath. The pH level on an ABG decreases as the episode is prolonged, but is increased early in the exacerbation. With a life-threatening asthma attack, PEFR is 40% of the patient's personal best.

The nurse is educating the patient on oxygen use for the home. The patient asks the nurse "Does this mean that I will not be able to go anywhere?" The most appropriate response by the nurse is:

Encourage the patient who uses home O2 to continue normal activity and to travel normally; this helps the patient maintain quality of life. Explaining the need to minimize activity in the home to conserve oxygen use, pointing out that distance traveling may not be possible because oxygen tanks are so small, and pointing out that most travel companies do not accommodate travelers with oxygen are incorrect because minimizing activity in the home most likely would decrease the patient's mobility and lead to other health conditions. If travel is by automobile, arrangements can be made for O2 to be available at the destination point. O2 supply companies often can assist in these arrangements. If a patient wishes to travel by bus, train, or airplane, the patient should inform the appropriate people when reservations are made that O2 will be needed for travel.

On examining a patient with asthma the nurse finds that the patient experiences asthmatic symptoms throughout the day, besides experiencing night-time awakenings more than four times a week. The patient's forced expiratory volume in the first second of expiration (FEV1) is less than 60%, and normal activity is very limited. Which treatment option should the nurse consider appropriate?

Experiencing symptoms of asthma throughout the day and also experiencing nighttime awakenings more than four times a week is suggestive of poorly controlled asthma. This patient also has an FEV1 less than 60%; normal activity being very limited correlates with that. Treatment with oral corticosteroids should be considered. Advice for maintaining control of asthma symptoms and following up after a month can be given in case of well controlled asthma. Reevaluation in two to six weeks is suggested if the asthma is not well controlled or if the patient experiences symptoms more than two times a month

A patient with chronic obstructive pulmonary disease (COPD) needs to be taught about effective huff coughing in a stepwise manner. In which order should the nurse put the steps for teaching the patient?

For effective huff coughing, the patient should assume a sitting position with his head slightly flexed, shoulders relaxed, knees flexed, forearms supported by a pillow, and, if possible, feet on the floor. Then the patient should inhale slowly through the mouth while breathing deeply from the diaphragm; the patient should hold the breath for two to three seconds. The patient should forcefully exhale quickly, as if fogging up a mirror with his breath to create a "huff," which moves the secretions to larger airways.

A patient with asthma is prescribed formoterol. What should the nurse include in the medication education provided to the patient? Select all that apply

Formoterol is a long-acting β2-adrenergic agonist (LABA) and is effective in treating symptoms of asthma that persist at night. Formoterol is not helpful to treat wheezing that gets worse, because short-acting beta blockers may be more beneficial. Side effects include cold or flu-like symptoms. LABAs should never be used as monotherapy for asthma, and they should only be used if the patient is taking inhaled corticosteroids. Short-acting beta blockers help the patient to obtain quick relief from bronchospasm. The patient must take the drug every 12 hours. The nurse should instruct the patient not to overuse formoterol, because there are other options also available.

The nurse would monitor which comorbidity in the patient treated for an asthma exacerbation with methylprednisolone?

Hyperglycemia or increased blood glucose level is an adverse effect of methylprednisolone, so the patient with diabetes mellitus should be monitored for elevations in blood sugar. Methylprednisolone will not affect elevated cholesterol, hypothyroidism, or Raynaud's phenomenon

A patient with late-stage chronic obstructive pulmonary disease (COPD) presented to the emergency room with increasing dyspnea. Lab results indicate a pH of 7.36, a PaCO2 of 47 mm Hg, and a HCO3 of 25 mEq/L. The patient asks the nurse what these lab results mean. What does the nurse know this lab data indicates?

In the patient with late-stage COPD, a low-normal pH, a high-normal or above normal PaCO2, and a high-normal HCO3 indicate compensated respiratory acidosis. This occurs when CO2 is chronically retained and the kidneys compensate to increase the pH to normal range. Severe hypoxemia and respiratory failure can both be related to COPD, however these lab results in the late stage of COPD indicate compensated respiratory acidosis. The pH is high-normal (normal range 7.35-7.45), the PaCO2 is above normal (normal range 35-45 mm Hg), the HCO3 is high normal (normal range 22-26 mEq/L); thus the lab values are not all within the normal range.

The nurse recalls that the steps that lead to airway remodeling occur in what order?

Inhaling animal dander triggers the inflammatory cascade in the patient. As the inflammatory process begins, the degranulation of mast cells takes place, leading to the release of inflammatory mediators. Then, inflammatory cells such as macrophages, eosinophils, and neutrophils infiltrate the airway to produce an inflammatory process. This results in neuropeptide release, with autonomic nervous system side effects that ultimately result in airway remodeling.

A patient presents to the emergency department with acute exacerbation of asthma. What actions should the nurse perform to monitor the patient's respiratory and cardiovascular systems? Select all that apply.

It is essential to monitor respiratory and cardiovascular systems in case of acute exacerbation of asthma. Auscultating lung sounds, measuring blood pressure and respiratory rate, and monitoring ABGs and pulse oximetry are required to monitor these systems. Chest radiographs are seldom useful in the management of an acute asthma attack. Checking the temperature may not contribute to monitoring respiratory and cardiovascular systems.

The nurse is educating a patient who was recently diagnosed with asthma about the use of a peak flow meter. Which statement is the nurse's priority?

It is important that the patient identify his or her personal best readings to evaluate when the symptoms are evolving into a medical emergency. The peak flow meter does not reduce asthma attacks; it helps monitor symptoms of asthma. It should be used at least twice a day for the first two weeks to determine the patient's personal best, which will be used to monitor airway constriction. The peak flow meter is used after the use of an inhaler, not before, to measure the effectiveness of the medication.

While teaching a patient with asthma about the appropriate use of a peak flow meter, what should the nurse instruct the patient to do?

It is important to keep track of peak flow readings daily, especially when the patient's symptoms are getting worse. The patient should have specific directions as to when to call the health care provider based on personal peak flow numbers. Peak flow is measured by exhaling into the flow meter, and should be assessed before and after medications to evaluate their effectiveness.

Which test result identifies that a patient with asthma is responding to treatment?

Nitric oxide levels are increased in the breath of people with asthma. A decrease in the exhaled nitric oxide concentration suggests that the treatment may be decreasing the lung inflammation associated with asthma and adherence to treatment. An increase in CO2 levels, decreased white blood cell count, and increased serum bicarbonate levels do not indicate a positive response to treatment in the asthma patient.

A patient with allergic asthma has been prescribed omalizumab. The patient requests that the medication be administered at home for convenience. Which nursing action is appropriate in this case?

Omalizumab is a monoclonal antibody to IgE that decreases circulating free IgE levels. The drug prevents IgE from attaching to mast cells, preventing the release of chemical mediators that may exacerbate asthma. The medication can cause anaphylactic reaction and should be administered at a clinic that is well-equipped to handle emergencies. The nurse should not encourage self-administration of the medication at home, because the anaphylactic reaction can be life-threatening. The nurse may not be well equipped to handle emergencies at home, so the medication should not be administered during a home visit. A family member should not be asked to administer the medication for the same reason.

When teaching a patient with chronic obstructive pulmonary disease (COPD) about using pursed-lip breathing (PLB), what is the most important instruction?

PLB should be used before, during, and after activities that cause shortness of breath. The correct way of doing PLB is to exhale by relaxing the facial muscles without puffing the cheeks and to inhale slowly through the nose. Exhalation should be made three times longer than inhalation. During PLB, inhalation should be done slowly and deeply through the nose.

Pulmonary rehabilitation (PR) is an evidence-based intervention that includes many disciplines working together to individualize treatment of the symptomatic chronic obstructive pulmonary disease (COPD) patient. What is PR designed to do? Select all that apply.

PR is an evidence-based intervention that includes many disciplines working together to individualize treatment of the symptomatic COPD patient. PR is designed to reduce symptoms and improve quality of life. PR is an effective intervention to improve exercise capacity and decrease hospitalizations, anxiety, and depression. An energy-saving tip is to exhale when pushing, pulling, or exerting effort during an activity and inhale during rest. PR should no longer be viewed as a "last ditch" effort for patients with severe COPD.

The nurse is evaluating an asthmatic patient's knowledge of self-care. Which statement by the patient indicates an adequate understanding of the instructions given?

Part of a rescue plan for asthma patients is to have access to a short-acting bronchodilator, such as albuterol, to use for rapid control of symptoms. Asthma patients should get a flu shot annually, and the self-monitoring of one's "Personal Best" with a peak flow meter should be done at least daily as part of an asthma action plan. Corticosteroids cannot abort an asthma attack.

A patient with an acute attack of asthma is in a state of panic. Which nursing measures help to relieve the panic? Select all that apply.

Pursed-lip breathing keeps the airways open, slows down the respiratory rate, and encourages deep breathing. "Talking down" is a technique that helps to calm the patient. A calm, quiet, and reassuring nurse helps to pacify the patient. Use of sedatives should not be encouraged, because they may cause respiratory depression. There is no technique called "walking down."

The nurse is assigned to care for a patient in the emergency department admitted with an exacerbation of asthma. The patient has received a β2-adrenergic agonists bronchodilator and supplemental oxygen. If the patient's condition does not improve, the nurse should anticipate what as the most likely next step in treatment?

Systemic corticosteroids speed the resolution of asthma exacerbations and are indicated if the initial response to the β2-adrenergic agonists bronchodilator is insufficient. IV fluids may be used, but not to improve ventilation. Biofeedback therapy and pulmonary function testing may be used after recovery to assist the patient and monitor the asthma.

The nurse determines that a patient is experiencing the most common adverse effect of albuterol after noting which sign?

Tachycardia is a common adverse effect of the use of inhaled β2-adrenergic agonists because of its stimulant effect. Headache, diarrhea, and oral candidiasis are not associated adverse effects of albuterol.

The nurse is evaluating if a patient understands how to determine safely whether a metered dose inhaler (MDI) is empty. The nurse interprets that the patient understands this important information to prevent medication underdosing when the patient describes which method to check the inhaler?

The best method to determine when to replace an inhaler is by knowing the maximum puffs available per MDI and then replacing it after the number of days when those inhalations have been used (100 puffs/2 puffs each day = 50 days). It is no longer appropriate to see if a canister floats in water or not because this is not an accurate way to determine the remaining inhaler doses. Shaking the canister and checking the indicator on the side of the canister are not the most effective ways of determining whether an MDI is empty.

When teaching the patient with chronic obstructive pulmonary disease (COPD) about smoking cessation, what information should be included related to the effects of smoking on the lungs and the increased incidence of pulmonary infections?

The damage to the lungs includes alveolar macrophage dysfunction that increases the incidence of infections and thus increases patient discomfort and cost to treat the infections. Other lung damage that contributes to infections includes cilia paralysis or destruction, increased mucus secretion, and bronchospasms that lead to sputum accumulation and increased cough. The patient may be aware already of respiratory mucosa damage with hoarseness and decreased sense of smell and taste, but these do not increase the incidence of pulmonary infection

To promote airway clearance in a patient with pneumonia and asthma, the nurse instructs the patient to perform which action?

The flutter device is used to increase mucus production to promote airway clearance and gas exchange; it should be used while the patient is in an upright position. Supplemental oxygen may not be indicated depending on the oxygen saturation level. Pursed-lip breathing and the incentive spirometer will not promote airway clearance.

The nurse concludes that interventions carried out to promote airway clearance in a patient admitted with asthma are successful on the basis of which finding?

The issue is airway clearance, which is evaluated most directly as successful if the patient can engage in effective and productive coughing. Oxygen saturation would indicate gas exchange, not airway clearance. Use of accessory muscles indicates respiratory distress. The absence of wheezing does not always coincide with improved airway clearance and may represent worsening bronchospasm.

What is a priority nursing assessment for a 38-year-old patient experiencing an acute asthma exacerbation?

The nurse physically inspects the chest wall to evaluate the use of intercostal muscles, which gives an indication of the degree of respiratory distress experienced by the patient. Pupillary response is a neurologic, not respiratory, assessment. Pedal pulses are measured to assess circulatory function. CVA tenderness is indicative of kidney inflammation, not asthma.

A patient who has been diagnosed with tuberculosis states, "I am not taking my medication, because I cannot afford it." Which ethical practices does the nurse demonstrate by implementing a social service consult to assist the patient in obtaining the medication needed for treatment? Select all that apply.

The nurse who includes other members of the health care team, such as social services, to assist in obtaining the necessary resources or support for the patient to complete a course of treatment is demonstrating patient and community advocacy. The social worker can facilitate the resources for the patient to obtain the medication but cannot ensure the patient's adherence to the treatment plan. Contacting a social worker does not provide a legal process for medication compliance. Provision of direct observed therapy involves providing the antituberculosis drugs directly to the patient and watching as he or she swallows the medications.

Which strategy should the nurse teach a patient with emphysema to manage anxiety related to impaired gas exchange?

The patient can decrease anxiety related to impaired gas exchange by performing pursed-lip breathing. The nurse cannot advise an increase in medication dosages without a health care provider's prescription. The patient should be encouraged to perform ADLs. Stress will not impact anxiety related to emphysema.

What does the nurse interpret from finding that a patient, after being treated for chronic cough and dyspnea associated with inflammation in lung parenchyma, loses muscle mass?

The patient has chronic cough and dyspnea associated with inflammation in lung parenchyma, indicating that the patient has chronic obstructive pulmonary disease (COPD). The patient experiences loss of muscle mass after treatment. This indicates that the patient is on high doses of corticosteroids, which may cause muscle loss due to protein catabolism. Diuretics are provided to reduce cardiac complications and may cause fluid imbalance. Theophylline is an antiinflammatory agent that helps to treat COPD effectively. Theophylline may cause side effects like bloating but is not associated with loss of muscle mass. Bronchodilators help to treat COPD by reducing the hyperinflation of lungs but are not associated with protein catabolism or muscle loss.

The nurse is teaching a patient how to use a hand-held nebulizer. Which guideline is correct?

The patient is placed in an upright position that allows for most efficient breathing to ensure adequate penetration and deposition of the aerosolized medication. The patient must breathe slowly and deeply through the mouth and hold inspirations for two or three seconds. Deep diaphragmatic breathing helps ensure deposition of the medication. Instruct the patient to breathe normally in between these large forced breaths to prevent alveolar hypoventilation and dizziness. After the treatment instruct the patient to cough effectively. An effective home-cleaning method is to wash the nebulizer equipment daily in soap and water, rinse it with water, and soak it for 20 to 30 minutes in a 1:1 white vinegar-water solution, followed by a water rinse and air drying.

The health care provider has prescribed an inhaled corticosteroid for the patient with asthma. The nurse should provide which instructions to the patient regarding the use of a dry powder inhaler (DPI)?

The patient should hold the breath for as long as possible to increase the amount of medication absorbed by the lungs. The DPI should not be shaken and should be stored in a cool area, and the patient's mouth should be closed tightly around the mouthpiece of the inhaler.

Which statement made by the patient with chronic obstructive pulmonary disease (COPD) indicates a need for further teaching regarding the use of an ipratropium inhaler?

The patient should not just keep taking extra puffs of the inhaler to make breathing easier. Excessive treatment could trigger paradoxical bronchospasm, which would worsen the patient's respiratory status. Rinsing the mouth after the puffs will eliminate a bad taste. Waiting one to two minutes between each puff will facilitate the effectiveness of the administration. Ipratropium is not used in an emergency for COPD

What is the best method to prevent oral infection while the patient is taking fluticasone?

The patient should rinse the mouth with water after the second puff of medication to reduce the risk of fungal overgrowth and oral infection. An oral antibiotic solution is not indicated and would not treat a fungal infection. Brushing the teeth is not necessary before medication administration and the mouth should be rinsed after, not before, medication administration.

The nurse is teaching a student nurse about breath sounds in a patient with asthma. Which statement made by the student nurse indicates the need for further teaching?

The patient with a minor attack may wheeze loudly during expiration, but the patient experiencing a severe asthma attack does not wheeze; he or she may have no audible sound on auscultation due to decreased airflow into the lungs. The patient must be able to move enough air through the airway to wheeze. Wheezing usually begins with exhalation and may be present during both inspiration and expiration as asthma progresses.

A patient in the emergency ward is dyspneic and speaks unclear words. Assessment findings include a respiratory rate of 45 breaths/minute, a pulse of 130 beats/minute, oxygen saturation of 90%, and neck vein distention. Which treatment does the nurse expect will help alleviate the patient's symptoms?

The patient with a severe attack of asthma has an elevated respiratory rate, decreased oxygen saturation and elevated pulse, and the inability to speak, which indicate a severe airway obstruction. The patient may also have neck vein distension. Hence the nurse should anticipate correction of hypoxemia and improve ventilation in the patient with supplementary oxygenation by nasal prongs. This helps to keep oxygen saturation above 90 percent. Administration of ipratropium does not provide additional benefits to the patient. Three puffs of albuterol every 30 minutes help to resolve the symptoms in the patients with mild asthma. Obtaining the peak flow rate and continuous monitoring of the patient is critical during asthma attack, but will not alleviate the patient's symptoms

Which assessment finding does the nurse expect when caring for a patient with asthma?

The patient with acute asthma may reveal signs of hypoxemia and hyperventilation due to air flow limitation, indicated by a low level of partial pressure of carbon dioxide in blood (PaCO2), such as 30 mmHg. This condition leads to a rise in pH leading to respiratory alkalosis; however, a pH of 5.11 is low. The respiratory rate of the asthmatic patient increases to more than 30 breaths/minute due to the use of accessory muscles. The patient with anxiety due to breathlessness has an increase in pulse and blood pressure.

What is the most appropriate time to obtain peak flow readings with the best peak flow number?

The peak flow readings should be taken between noon and 2 PM because the peak flow is highest during this period. The patient may feel nauseous if the peak flow readings are taken five minutes after meals. The peak flow readings are not at a high level early in the morning. The peak flow readings should be taken 20 minutes after administering short-acting β2-agonist because it is more effective; the medication would not be very effective two hours after inhaling a short-acting β2-agonist

The nurse teaches a 61-year-old patient with emphysema and pneumonia to obtain which vaccines? Select all that apply.

The pneumococcal and influenza vaccines are important for patients with a history of heart or lung disease and those recovering from a severe illness, age 65 or over, or living in a long-term care facility. Tetanus, BCG, and hepatitis B vaccines are not recommended for patients to protect the lungs from additional damage, although they may be required for other reasons.

Which complications, along with edema in the ankles, does the nurse expect in a patient with chronic obstructive pulmonary disease (COPD)? Select all that apply.

The presence of edema in the ankles indicates that the patient has the cardiac complication cor pulmonale, or failure in the right side of the heart. Cor pulmonale is associated with peripheral edema, especially in the ankles, so the patient will have weight gain. The patient with cor pulmonale will have tissue damage, resulting in hepatomegaly. The patient with cor pulmonale will have improper blood flow from the right atrium to the right ventricle, resulting in jugular vein distension. The patient with COPD will not have a decrease in white blood cells. The patient with COPD will have increased red blood cells due to hypoxemia, but will not have polycythemia.

The nurse is explaining the pathophysiology of asthma to a patient. Which is the most appropriate explanation?

The primary pathophysiologic process in asthma is persistent but variable inflammation of the airways. The airflow is limited because the inflammation results in bronchoconstriction, airway hyperresponsiveness (hyperreactivity), and edema of the airways. Exposure to allergens or irritants initiates the inflammatory cascade. An immune response does not trigger asthma. Inflammation causes edema, not pressure, of the airways. Acid-base imbalances do not trigger asthma.

The nurse administering beclomethasone to a patient can help reduce side effects by instructing the patient to perform which action?

The side effects of the glucocorticoid include oral candida, which can be prevented through rinsing the mouth with water or mouthwash after each use. This medication should be used on a schedule, not as needed, a spacer is encouraged, and it will not be helpful at the onset of an asthma attack because it is not an abortive medication; it is a preventative medication.

The patient with COPD has a prescription for ibuprofen 600 mg. The nurse questions the prescription, knowing that ibuprofen interacts with which concurrent medication?

The use of ibuprofen should be questioned with the concurrent use of hydrocortisone, a 28 corticosteroid. The use of these medications together increases the risk of gastrointestinal bleeding. Ibuprofen and sertraline, azithromycin, or ipratropium will not cause adverse effects

A patient develops vascular congestion, edema, and breathlessness within 30 minutes of the consumption of food containing tartrazine. The nurse identifies that which treatment may exacerbate the patient's condition?

Vascular congestion, edema, and breathlessness within 30 minutes indicate the early-phase response in asthma. Consumption of food containing tartrazine can trigger the inflammatory process. The patient with such a condition should avoid intravenous administration of corticosteroids, because they contain sulfite-containing preservatives, which further aggravate the condition. Administration of anticholinergics is safe in initial stages of asthma. Assisted ventilation and inhalation of oxygen by mask helps to treat life-threatening attacks of asthma.

The nurse is performing chest physiotherapy on a patient with chronic obstructive pulmonary disease. What appropriate techniques should be used when performing the procedure on this patient? Select all that apply.

Vibration, percussion, and postural drainage are chest physiotherapy techniques that help to remove excessive secretions in the lungs. Huff coughing and pursed-lip breathing are breathing techniques, which also help in the movement of chest secretions. Huff coughing and pursed-lip breathing are performed before chest physiotherapy but are not considered to be chest physiotherapy.

An asthmatic patient is in acute respiratory distress. The nurse auscultates the lungs and notes cessation of inspiratory wheezing. How does the nurse interpret this finding?

When the patient in respiratory distress has inspiratory wheezing that then ceases, it is an indication of airway obstruction and requires emergency action to restore the airway. A pneumothorax would be evidenced by absent breath sounds. Absence of wheezing does not correlate with improved airflow if the patient is also in current respiratory distress. A mucus plug would result in crackles in the lungs.

A nurse observes a patient with asthma using a metered-dose inhaler. The nurse should correct which patient actions? Select all that apply

While using a metered dose inhaler, the breath should be held for at least 10 seconds for better absorption of the drug. The metered dose inhaler should be pressed with adequate strength to deliver the required dose. Inspiring slowly, shaking the device before using it, and coordinating use of the inhaler with inspiration are the right techniques for using a metered dose inhaler.


Kaugnay na mga set ng pag-aaral

EMT Chapter 30 - Abdominal Genitourinary Injuries

View Set

Ohio Life insurance missed practice test questions and answers part 3

View Set

Participatory Budgeting (PB) and the Participatory Budgeting Project (PBP)

View Set

Chapter 4: Cultural & Developmental Processes

View Set